Med surg final exam review practice questions FINAL HOLY SHIT THIS IS LONG

¡Supera tus tareas y exámenes ahora con Quizwiz!

Which instruction about insulin administration should a nurse give to a client? "Discard the intermediate-acting insulin if it appears cloudy." "Always follow the same order when drawing the different insulins into the syringe." "Shake the vials before withdrawing the insulin." "Store unopened vials of insulin in the freezer at temperatures well below freezing."

"Always follow the same order when drawing the different insulins into the syringe."

A client with type 1 diabetes has a highly elevated glycosylated hemoglobin (Hb) test result. In discussing the result with the client, the nurse is most accurate in stating: "The test must be repeated following a 12-hour fast." "It tells us about your sugar control for the last 3 months." "Your insulin regimen must be altered significantly." "It looks like you aren't following the ordered diabetic diet."

"It tells us about your sugar control for the last 3 months."

A client with diabetes mellitus must learn how to self-administer insulin. The physician has ordered 10 units of U-100 regular insulin and 35 units of U-100 isophane insulin suspension (NPH) to be taken before breakfast. When teaching the client how to select and rotate insulin injection sites, the nurse should provide which instruction? "Rotate injection sites within the same anatomic region, not among different regions." "Administer insulin into areas of scar tissue or hypertrophy whenever possible." "Inject insulin into healthy tissue with large blood vessels and nerves." "Administer insulin into sites above muscles that you plan to exercise heavily later that day."

"Rotate injection sites within the same anatomic region, not among different regions."

Which instruction should a nurse give to a client with diabetes mellitus when teaching about "sick day rules"? "Don't take your insulin or oral antidiabetic agent if you don't eat." "Test your blood glucose every 4 hours." "Follow your regular meal plan, even if you're nauseous." "It's okay for your blood glucose to go above 300 mg/dl while you're sick."

"Test your blood glucose every 4 hours."

The client diagnosed with renal calculi is scheduled for a 24-hour urine specimen collection. Which interventions should the nurse implement? Select all that apply. 1. Check for the ordered diet and medication modifications. 2. Instruct the client to urinate, and discard this urine when starting collection. 3. Collect all urine during 24 hours and place in appropriate specimen container. 4. Insert a Foley catheter in client after having the client empty the bladder. 5. Post notices on the client's door to save all urine output.

***1. The health-care provider may order certain foods and medications when obtaining 24- hour urine collection to evaluate for calcium oxalate or uric acid. ***2. When the collection begins, the client should completely empty the bladder and discard that urine. ***3. All urine for 24 hours should be saved and put in a container with preservative, refrig- erated, or put on ice as indicated. Not following specific instructions will result in an inaccurate test result. 4. The urine is obtained in some type of urine collection device such as a bedpan, bedside commode, or commode hat. The client is not catheterized. ***5. Posting signs will help ensure that all the urine is saved during the 24-hour period. If any urine is discarded, the test may result in inaccurate information or the need to start the test over. TEST-TAKING HINT: This is an alternate-type question that may have more than one correct answer. The test taker must be knowledgeable of specific laboratory tests.

To help prevent infections in clients with COPD, the nurse should recommend vaccinations against two bacterial organisms. Which of the following are the two vaccinations? -Streptococcus pneumonia and Haemophilus influenzae -Streptococcus pneumonia and varicella -Haemophilus influenzae and varicella -Haemophilus influenzae and Gardasil

-Streptococcus pneumonia and Haemophilus influenzaeClients with COPD are more susceptible to respiratory infections, so they should be encouraged to receive the influenza and pneumococcal vaccines. Clients with COPD aren't at high risk for varicella or hepatitis B. The HPV vaccine is to guard against cervical cancer and is recommended only for females ages 9 to 26 years.

For a client with advanced chronic obstructive pulmonary disease (COPD), which nursing action best promotes adequate gas exchange? -Encouraging the client to drink three glasses of fluid daily -Keeping the client in semi-Fowler's position -Using a Venturi mask to deliver oxygen as ordered -Administering a sedative as ordered

-Using a Venturi mask to deliver oxygen as ordered

A client with chronic obstructive pulmonary disease (COPD) is admitted to the medical-surgical unit. To help this client maintain a patent airway and achieve maximal gas exchange, the nurse should: -instruct the client to drink at least 2 L of fluid daily. -maintain the client on bed rest. -administer anxiolytics, as ordered, to control anxiety. -administer pain medication as ordered.

-instruct the client to drink at least 2 L of fluid daily.

A client is receiving theophylline for long-term control and prevention of asthma symptoms. Client education related to this medication will include -the importance of blood tests to monitor serum concentrations. -taking the medication at least 1 hour prior to meals. -monitoring liver function studies as prescribed. -development of hyperkalemia.

-the importance of blood tests to monitor serum concentrations.The nurse should inform clients about the importance of blood tests to monitor serum concentration. The therapeutic range of theophylline is between 5 and 15 μg/mL. The client is at risk of developing hypokalemia.

A client is suspected of having cirrhosis of the liver. What diagnostic procedure will the nurse prepare the client for in order to obtain a confirmed diagnosis? 1- A liver biopsy 2- A CT scan 3- A prothrombin time 4- Platelet count

1

A client with urinary tract infection is prescribed phenazopyridine (Pyridium). Which of the following instructions would the nurse give the client? 1- "This medication will relieve your pain." 2- "This medication should be taken at bedtime." 3- "This medication will prevent re-infection." 4- "This will kill the organism causing the infection."

1

Patients with urolithiasis need to be encouraged to: 1- Increase their fluid intake so that they can excrete 2.5 to 4 liters every day. 2- Participate in strenuous exercises so that the tone of smooth muscle in the urinary tract can be strengthened to help propel calculi. 3- Supplement their diet with calcium needed to replace losses to renal calculi. 4- Limit their voiding to every 6 to 8 hours so that increased volume can increase hydrostatic pressure, which will help push stones along the urinary system.

1

The client diagnosed with IBD is prescribed total parental nutrition (TPN). Which intervention should the nurse implement? 1. Check the client's glucose level. 2. Administer an oral hypoglycemic. 3. Assess the peripheral intravenous site. 4. Monitor the client's oral food intake.

1

The client diagnosed with acute diverticulitis is complaining of severe abdominal pain. On assessment, the nurse finds a hard, rigid abdomen and T 102˚F. Which intervention should the nurse implement? 1. Notify the health-care provider. 2. Prepare to administer a Fleet's enema. 3. Administer an antipyretic suppository. 4. Continue to monitor the client closely.

1

The client with acute diverticulitis has a nasogastric tube draining green liquid bile. Which intervention should the nurse implement? 1. Document the findings as normal. 2. Assess the client's bowel sounds. 3. Determine the client's last bowel movement. 4. Insert the N/G tube at least 2 more inches.

1

The nurse is caring for an adult client diagnosed with GERD. Which condition is the most common comorbid disease associated with GERD? 1. Adult-onset asthma. 2. Pancreatitis. 3. Peptic ulcer disease. 4. Increased gastric emptying.

1

The nurse is educating a patient with urolithiasis about preventive measures to avoid another occurrence. What should the patient be encouraged to do? 1- Increase fluid intake so that the patient can excrete 2,500 to 4,000 mL every day, which will help prevent additional stone formation. 2- Participate in strenuous exercises so that the tone of smooth muscle in the urinary tract can be strengthened to help propel calculi. 3- Add calcium supplements to the diet to replace losses to renal calculi. 4- Limit voiding to every 6 to 8 hours so that increased volume can increase hydrostatic pressure, which will help push stones along the urinary system.

1

The nurse is employed in a urologist's office. Which classification of medication is anticipated for clients having difficulty with urinary incontinence? 1- Anticholinergic 2- Diuretics 3- Anticonvulsant 4- Cholinergic

1

The nurse is encouraging the client with recurrent urinary tract infections to increase his fluid intake to 8 large glasses of fluids daily. The client states he frequently drinks water and all of the following. Which of the following would the nurse discourage for this client? 1- Coffee in the morning 2- Fruit juice midmorning 3- Milk at lunch 4- Ginger ale at dinner time

1

Which data should the nurse expect to assess in the client who had an upper gastrointestinal (UGI) series? 1. Chalky white stools. 2. Increased heart rate. 3. A firm hard abdomen. 4. Hyperactive bowel sounds.

1

Which of the following is a cause of a calcium renal stone? 1- Excessive intake of vitamin D 2- Gout 3- Neurogenic bladder 4- Foreign bodies

1

Which of the following types of cardiomyopathy can be associated with childbirth? 1. Dilated 2. Hypertrophic 3. Myocarditis 4. Restrictive

1

Which priority teaching information should the nurse discuss with the client to help prevent contracting hep. A? 1.Explain the importance of good hand washing. 2.Tell the client to take the hepatitis B vaccine in three (3) doses. 3.Tell the client not to ingest unsanitary food or water. 4.Discuss how to implement standard precautions.

1

Which sign/symptom should the nurse expect to find in a client diagnosed with ulcerative colitis? 1. Twenty bloody stools a day. 2. Oral temperature of 102˚F. 3. Hard, rigid abdomen. 4. Urinary stress incontinence.

1

Which type of voiding dysfunction is seen in clients diagnosed with Parkinson disease? 1- Incontinence 2- Urinary retention 3- Urgency 4- Incomplete bladder emptying

1

The client diagnosed with Guillain-Barré syndrome asks the nurse, "Will I ever get back to normal?I am so tired of being sick." Which statement is the best response by the nurse? 1. "You should make a full recovery within a few months to a year." 2. "Most clients with this syndrome have some type of residual disability." 3. "This is something you should discuss with the health-care team." 4. "The rehabilitation is short and you should be fully recovered within a month."

1 Clients with this syndrome usuallyhave a full recovery, but it may take upto one (1) year. 2. Only about 10% of clients are left with permanent residual disability. 3. This is "passing the buck." The nurse should answer the client's question honestly, which helps establish a trusting nurse-client relationship. 4. This indicates the nurse does not under-stand the typical course for a client diagnosed with Guillain-Barré syndrome.

Which assessment intervention should the nurse implement specifically for the diagnosis of Guillain-Barré syndrome? 1. Assess deep tendon reflexes. 2. Complete a Glasgow Coma Scale. 3. Check for Babinski's reflex. 4. Take the client's vital signs.

1 Hyporeflexia of the lower extremities is the classic clinical manifestation of this syndrome. Therefore, assessing deep tendon reflexes is appropriate. 2. A Glasgow Coma Scale is used for clients with potential neurological deficits and used to monitor for increased intracranial pressure. 3. Babinski's reflex evaluates central nervous system neurological status, which is not affected with this syndrome. 4. Vital signs are a part of any admission assessment but are not a specific assessment intervention for this syndrome

The nurse is teaching a class on diverticulosis. Which interventions should the nurse discuss when teaching ways to prevent an acute exacerbation of diverticulosis? Select all that apply. 1. Eat a high-fiber diet. 2. Increase fluid intake. 3. Elevate the HOB after eating. 4. Walk 30 minutes a day. 5. Take an antacid every two (2) hours.

1, 2, 4

The client diagnosed with Guillain-Barré syndrome is admitted to the rehabilitation unit after 23 days in the acute care hospital. Which interventions should the nurse implement?Select all that apply. 1. Refer client to the physical therapist. 2. Include the speech therapist in the team. 3. Request a social worker consult. 4. Implement a regimen to address pain control. 5. Refer the client to the Guillain-Barré Syndrome Foundation

1, 3, 4, 5 1.The physical therapist is an important part of the rehabilitation team who addresses the client's muscle deterioration resulting from the disease process and immobility. 3.The social worker could help with financial concerns, job issues, and issues concerning the long rehabilitation time for this syndrome. 4.Pain may or may not be an issue with this syndrome. Each client is different,but a plan needs to be established to address pain if it occurs. 5.This is an excellent resource for the client and the family 2. There is no residual speech deficit fromGuillain-Barré syndrome; therefore, this referral is not appropriate.

A client is hospitalized with a diagnosis of sickle cell crisis. Which findings should lead a nurse to conclude that outcomes have been achieved for this client? SELECT ALL THAT APPLY. 1. Leukocyte count 7,500/mm3 2. Describes the importance of keeping warm 3. Acute pain controlled at less than 3 on a 0 to 10 scale with analgesics 4. Free of chest pain or dyspnea 5. Blood transfusions effective in diminishing cell sickling 6. Hydroxyurea (Hydrea®) effective in suppressing leukocyte formation

1,2,3,4

The client has a hematocrit of 22.3% and a hemoglobin of 7.7 g/dL. The HCP has ordered two (2) units of packed red blood cells to be transfused. Which interventions should the nurse implement? Select all that apply. 1. Obtain a signed consent. 2. Initiate a 22-gauge IV. 3. Assess the client's lungs. 4. Check for allergies. 5. Hang a keep-open IV of D5W

1,3,4

The client is being admitted with folic acid deficiency anemia. Which would be the most appropriate referral? 1. Alcoholics Anonymous. 2. Leukemia Society of America. 3. A hematologist. 4. A social worker.

1. Alcoholics Anonymous. Most clients diagnosed with folic acid deficiency anemia have developed the anemia from chronic alcohol abuse. Alcohol consumption increases the use of folates, and the alcoholic diet is usually deficient in folic acid. A referral to Alcoholics Anonymous would be appropriate

The client is diagnosed with a uric acid stone. Which foods should the client eliminate from the diet to help prevent reoccurrence? 1. Beer and colas. 2. Asparagus and cabbage. 3. Venison and sardines. 4. Cheese and eggs.

1. Beer and colas are foods high in oxalate, which can cause calcium oxalate stones. 2. Asparagus and cabbage are foods high in oxalate, which can cause calcium oxalate stones. ***3. Venison, sardines, goose, organ meats, and herrings are high purine foods, which should be eliminated from the diet to help prevent uric acid stones. 4. Cheese and eggs are foods that help acidify the urine and do not cause the development of uric acid stones. TEST-TAKING HINT: The nurse has to be knowledgeable of foods included in specific diets. This is memorizing, but the reader must have this knowledge to answer questions evaluating types of diets for specific diseases and disorders.

Which clinical manifestations would the nurse expect to assess for the client diagnosed with a ureteral renal stone? 1. Dull, aching flank pain and microscopic hematuria. 2. Nausea; vomiting; pallor; and cool, clammy skin. 3. Gross hematuria and dull suprapubic pain with voiding. 4. No symptoms.

1. Dull flank pain and microscopic hematuria are manifestations of a renal stone in the kidney. ***2. The severe flank pain associated with a stone in the ureter often causes a sympathetic response with associated nausea; vomiting; pallor; and cool, clammy skin. 3. Gross hematuria and suprapubic pain when voiding are manifestations of a stone in the bladder. 4. Kidney stones and bladder stones may produce no signs/symptoms, but a ureteral stone always causes pain on the affected side because a ureteral spasm occurs when the stone obstructs the ureter. TEST-TAKING HINT: Note that options "1" and "3" both have assessment data that indicate bleeding. The test taker can usually eliminate these as possible answers or eliminate the other two options that do not address blood. Renal stones are painful; therefore "4" could be eliminated as a possible answer.

The nurse writes a client problem of "activity intolerance" for a client diagnosed with anemia. Which intervention should the nurse implement? 1. Pace activities according to tolerance. 2. Provide supplements high in iron and vitamins. 3. Administer packed red blood cells. 4. Monitor vital signs every four (4) hours.

1. Pace activities according to tolerance.

The client with O+ blood is in need of an emergency transfusion but the laboratory does not have any O+ blood available. Which potential unit of blood could be given to the client? 1. The O- unit. 2. The A+ unit. 3. The B+ unit. 4. Any Rh+ unit.

1. The O- unit.

The charge nurse in the intensive care unit is making client assignments. Which client should the charge nurse assign to the graduate nurse who has just finished the three (3)-month orientation? 1. The client with an abdominal peritoneal resection who has a colostomy. 2. The client diagnosed with pneumonia who has acute respiratory distress syndrome. 3. The client with a head injury developing disseminated intravascular coagulation. 4. The client admitted with a gunshot wound who has an H&H of 7 and 22.

1. The client with an abdominal peritoneal resection who has a colostomy.

76. The nurse and the unlicensed assistive personnel (UAP) are caring for clients on a medical-surgical unit. Which task should not be assigned to the UAP? 1. Feed the 69-year-old client diagnosed with Parkinson's disease who is having difficulty swallowing. 2. Turn and position the 89-year-old client diagnosed with a pressure ulcer secondary to Parkinson's disease. 3. Assist the 54-year-old client diagnosed with Parkinson's disease with toilet-training activities. 4. Obtain vital signs on a 72-year-old client diagnosed with pneumonia secondary to Parkinson's disease.

1. The nurse should not delegate feeding a client who is at risk for complications during feeding. This requires judgment that the UAP is not expected to possess.

The laboratory data reveal a calcium phosphate renal stone for a client diagnosed with renal calculi. Which discharge teaching intervention should the nurse implement? 1. Encourage the client to eat a low-purine diet and limit foods such as organ meats. 2. Explain the importance of not drinking water two (2) hours before bedtime. 3. Discuss the importance of limiting vitamin D-enriched foods. 4. Prepare the client for extracorporeal shock wave lithotripsy (ESWL).

1. This would be appropriate for the client who has uric acid stones. 2. The nurse should recommend drinking one to two glasses of water at night to prevent concentration of urine during sleep. ***3. Dietary changes for preventing renal stones include reducing the intake of the primary substance forming the calculi. In this case, limiting vitamin D will inhibit the absorption of calcium from the gastrointestinal tract. 4. This is a treatment for an existing renal stone, not a discharge teaching intervention for a client who has successfully passed a renal calculus. TEST-TAKING HINT: Remember to read the question carefully. The question asks for a "discharge teaching" intervention. This would rule out "4," which is a treatment, as a potential answer.

The nurse is admitting a 24-year-old African American female client with a diagnosis of rule-out anemia. The client has a history of gastric bypass surgery for obesity four (4) years ago. Current assessment findings include height 5′5′′; weight 75 kg; P 110, R 27, and BP 104/66; pale mucous membranes and dyspnea on exertion. Which type of anemia would the nurse suspect the client has developed? 1. Vitamin B12 deficiency. 2. Folic acid deficiency. 3. Iron deficiency. 4. Sickle cell anemia.

1. Vitamin B12 deficiency.

2. A client with rheumatoid arthritis states, "I can't do my household chores without becoming tired. My knees hurt whenever I walk." Which nursing diagnosis would be most appropriate? 1. Activity intolerance related to fatigue and pain. 2. Self-care deficit related to increasing joint pain. 3. Ineffective coping related to chronic pain. 4. Disturbed body image related to fatigue and joint pain.

1.Based on the client's complaints, the most appropriate nursing diagnosis would be Activity intolerance related to fatigue and pain. Nursing interventions would focus on helping the client conserve energy and decrease episodes of fatigue. Although the client may develop a self-care deficit related to the activity intolerance and increasing joint pain, the client is voicing concerns about household chores and difficulty around the house and yard, not self-care issues. Over time, the client may develop ineffective coping or body image disturbance as the disorder becomes chronic with increasing pain and fatigu

A nurse is caring for a client with diabetes mellitus. The client has a blood glucose level of 40 mg/dL. Which of the following rapidly absorbed carbohydrate would be most effective? 1/2 cup fruit juice or regular soft drink Three to six LifeSavers candies 1/2 tbsp honey or syrup 4 oz of skim milk

1/2 cup fruit juice or regular soft drink

The nurse is administering lispro (Humalog) insulin. Based on the onset of action, how soon should the nurse administer the injection prior to breakfast? 10 to 15 minutes 1 to 2 hours 30 to 40 minutes 3 hours

10 to 15 minutes

A nurse is preparing a continuous insulin infusion for a child with diabetic ketoacidosis and a blood glucose level of 800 mg/dl. Which solution is the most appropriate at the beginning of therapy? 100 units of NPH insulin in dextrose 5% in water 100 units of regular insulin in dextrose 5% in water 100 units of regular insulin in normal saline solution 100 units of neutral protamine Hagedorn (NPH) insulin in normal saline solution

100 units of regular insulin in normal saline solution

Of the clients listed below, who is at risk for developing rheumatoid arthritis (RA)? Select all that apply. 1. Adults between the ages of 20 and 50 years. 2. Adults who have had an infectious disease with the Epstein-Barr virus. 3. Adults that are of the male gender. 4. Adults who possess the genetic link, specifically HLA-DR4. 5. Adults who also have osteoarthritis.

124

A nurse is teaching a client with diabetes mellitus about self-management of his condition. The nurse should instruct the client to administer 1 unit of insulin for every: 20 g of carbohydrates. 25 g of carbohydrates. 10 g of carbohydrates. 15 g of carbohydrates.

15 g of carbohydrates.

A 49-year-old man with a history of heavy alcohol use and liver cirrhosis has been admitted to the hospital's medical unit due to an exacerbation of his health problems that has resulted in massive ascites. The nurse should be prepared to implement which of the following interventions in an effort to resolve the patient's ascites? 1- High Fowler's positioning and fluid restriction 2- Low-sodium diet and administration of diuretics 3- Peritoneal massage and administration of hypertonic IV solutions 4- Low-protein diet and administration of IV albumin

2

A 69-year-old man is postoperative day 2 following a transurethral prostatic resection (TUPR). The patient had his urinary catheter removed at 06:00 this morning but has not voided in the 5 hours since the removal, despite the fact that he has been drinking large amounts of fluids. What nursing assessment will most accurately determine whether the patient is retaining urine? 1- Bladder palpation 2- Bladder ultrasound 3- Inspection of the patient's pubic region 4- An audit of the patient's recent intake and output

2

A client comes to the clinic for a follow-up visit. During the interview, the client states, "Sometimes when I have to urinate, I can't control it and do not reach the bathroom in time." The nurse suspects that the client is experiencing which type of incontinence? 1- Stress 2- Urge 3- Overflow 4- Functional

2

A client has just been diagnosed with hepatitis A. On assessment, the nurse expects to note: 1- severe abdominal pain radiating to the shoulder. 2- anorexia, nausea, and vomiting. 3- eructation and constipation. 4- abdominal ascites.

2

A client is prescribed amitriptyline (an antidepressant) for incontinence. The nurse understands that this drug is an effective treatment because it: 1- increases contraction of the detrusor muscle. 2- increases bladder neck resistance. 3- reduces bladder spasticity. 4- decreases involuntary bladder contractions.

2

Examination of a client's bladder stones reveal that they are primarily composed of uric acid. The nurse would expect to provide the client with which type of diet? 1- Low oxalate 2- Low purine 3- High protein 4- High sodium

2

The client diagnosed with ulcerative colitis has an ileostomy. Which statement indicates the client needs more teaching concerning the ileostomy? 1. "My stoma should be pink and moist." 2. "I will irrigate my ileostomy every morning." 3. "If I get a red, bumpy, itchy rash I will call my HCP." 4. "I will change my pouch if it starts leaking."

2

The client is diagnosed with an acute exacerbation of ulcerative colitis. Which intervention should the nurse implement? 1. Provide a low-residue diet. 2. Rest the client's bowel. 3. Assess vital signs daily. 4. Administer antacids orally.

2

The male client tells the nurse he has been experiencing "heartburn" at night that awakens him. Which assessment question should the nurse ask? 1. "How much weight have you gained recently?" 2. "What have you done to alleviate the heartburn?" 3. "Do you consume many milk and dairy products?" 4. "Have you been around anyone with a stomach virus?"

2

The nurse advises a patient with renal stones to avoid eating shellfish, asparagus, and organ meats. She emphasizes these foods because she knows that his renal stones are composed of which of the following substances? 1- Calcium 2- Uric acid 3- Struvite 4- Cystine

2

The nurse is educating a female patient with a UTI on the pharmacologic regimen for treatment. What is important for the nurse to instruct the patient to do? 1- Take the antibiotic as well as an antifungal for the yeast infection she will probably have. 2- Take the antibiotic for 3 days as prescribed. 3- Understand that if the infection reoccurs, the dose will be higher next time. 4- Be sure to take the medication with grapefruit juice.

2

The nurse recognizes that urinalysis results that most likely indicate a urinary tract infection include: 1- proteinuria 2- WBC 50 3- RBC 3 4- glucose trace

2

When caring for a client with advanced cirrhosis and hepatic encephalopathy, which assessment finding should the nurse report immediately? 1- Weight loss of 2 pounds in 3 days 2- Change in the client's handwriting and/or cognitive performance 3- Anorexia for more than 3 days 4- Constipation for more than 2 days

2

Which statement made by the client indicates to the nurse the client may be experiencing GERD? 1. "My chest hurts when I walk up the stairs in my home." 2. "I take antacid tablets with me wherever I go." 3. "My spouse tells me I snore very loudly at night." 4. "I drink six (6) to seven (7) soft drinks every day."

2

The client diagnosed with Guillain-Barré syndrome is on a ventilator. Which intervention will assist the client to communicate with the nursing staff? 1. Provide an erase slate board for the client to write on .2. Instruct the client to blink once for "no" and twice for "yes." 3. Refer to a speech therapist to help with communication. 4. Leave the call light within easy reach of the client.

2 The client will not be able to use the arms as a result of the paralysis but can blink the eyes as long as the nurse asks simple "yes-or-no" questions. 1.The ascending paralysis has reached the client's respiratory muscles; therefore, the client will not be able to use the hands to write. 3. A speech therapist will not be able to help the client communicate while the client is on the ventilator. 4. The ascending paralysis has reached the respiratory muscles; therefore, the client will not be able to use the hands to push the call light

The nurse caring for the client diagnosed with Guillain-Barré syndrome writes the client problem "impaired physical mobility." Which long-term goal should be written for this problem? 1. The client will have no skin irritation. 2. The client will have no muscle atrophy. 3. The client will perform range-of-motion exercises. 4. The client will turn every two (2) hours while awake.

2 The client with Guillain-Barré syndrome will not be able to move the extremities; therefore, preventingmuscle atrophy is an appropriate long-term goal 1. This is an appropriate long-term goal for the client problem "impaired skin integrity." 3. The client will not be able to move the extremities. Therefore, the nurse will have to do passive range-of-motion exercises;this is an intervention, not a goal. 4. This is a nursing intervention, not a goal,and the client should be turned while sleeping unless the client is on a special immobility bed

Which statement by the client supports the diagnosis of Guillain-Barré syndrome? 1. "I just returned from a short trip to Japan." 2. "I had a really bad cold just a few weeks ago." 3. "I think one of the people I work with had this." 4. "I have been taking some herbs for more than a year."

2 This syndrome is usually preceded by a respiratory or gastrointestinal infection one (1) to four (4) weeks prior to the onset of neurological deficits. 1. Visiting a foreign country is not a risk factor for contracting this syndrome. 3. This syndrome is not a contagious or a communicable disease. 4. Taking herbs is not a risk factor for developing Guillain-Barré syndrome

The nurse is providing an education program for the nursing assistants in a long-term care facility in order to decrease the number of UTIs in the female population. What interventions should the nurse introduce in the program? Select all that apply. 1- For those patients who are incontinent, insert indwelling catheters. 2- Perform hand hygiene prior to patient care. 3- Assist the patients with frequent toileting. 4- Provide careful perineal care. 5- Encourage patients to wear briefs.

2,3,4

The health-care provider scheduled a lumbar puncture for a client admitted with rule-out Guillain-Barré syndrome. Which pre procedure intervention has priority? 1. Keep the client NPO. 2. Instruct the client to void. 3. Place in the lithotomy position. 4. Assess the client's pedal pulse.

2. The client should void prior to this procedure to help prevent accidental puncture of the bladder during the procedure. 1. The client does not need to be NPO prior to this procedure. 3. The lithotomy position has the client lying flat with the legs in stirrups, such as when Pap smears are obtained. 4. The pedal pulses should be assessed postprocedure, not prior to the procedure.

The client's nephew has just been diagnosed with sickle cell anemia. The client asks the nurse, "How did my nephew get this disease?" Which statement would be the best response by the nurse? 1. "Sickle cell anemia is an inherited autosomal recessive disease." 2. "He was born with it and both his parents were carriers of the disease." 3. "At this time, the cause of sickle cell anemia is unknown." 4. "Your sister was exposed to a virus while she was pregnant."

2. "He was born with it and both his parents were carriers of the disease."

The nurse is completing discharge teaching for the client diagnosed with a sickle cell crisis. The nurse recommends the client getting the flu and pneumonia vaccines. The client asks, "Why should I take those shots? I hate shots." Which statement by the nurse is the best response? 1. "These vaccines promote health in clients with chronic illnesses." 2. "You are susceptible to infections. These shots may help prevent a crisis." 3. "The vaccines will help your blood from sickling secondary to viruses." 4. "The doctor wanted to make sure that I discussed the vaccines with you."

2. "You are susceptible to infections. These shots may help prevent a crisis."

The client diagnosed with sickle cell anemia comes to the emergency department complaining of joint pain throughout the body. The oral temperature is 102.4˚F and the pulse oximeter reading is 91%. Which action should the emergency room nurse implement first? 1. Request arterial blood gases STAT. 2. Administer oxygen via nasal cannula. 3. Start an IV with an 18-gauge angiocath. 4. Prepare to administer analgesics as ordered.

2. Administer oxygen via nasal cannula.

73. The client diagnosed with Parkinson's disease (PD) is being admitted with a fever and patchy infiltrates in the lung fields on the chest x-ray. Which clinical manifestations of PD would explain these assessment data? 1. Masklike facies and shuffling gait. 2. Difficulty swallowing and immobility. 3. Pill rolling of fingers and flat affect. 4. Lack of arm swing and bradykinesia.

2. Difficulty swallowing places the client at risk for aspiration. Immobility predisposes the client to pneumonia. Both clinical manifestations place the client at risk for pulmonary complications.

Which diagnostic test is used to confirm ALS? 1. Electromyelogram (EMG). 2. Muscle biopsy. 3. Serum creatine kinase enzyme (CK). 4. Pulmonary function test.

2. Muscle biopsy.

The nurse and an unlicensed assistive personnel (UAP) are caring for clients on a medical unit. Which task should the nurse delegate to the UAP? 1. Check on the bowel movements of a client diagnosed with melena. 2. Take the vital signs of a client who received blood the day before. 3. Evaluate the dietary intake of a client who has been noncompliant with eating. 4. Shave the client diagnosed with severe hemolytic anemia.

2. Take the vital signs of a client who received blood the day before.

Which statement is the scientific rationale for infusing a unit of blood in less than four (4) hours? 1. The blood will coagulate if left out of the refrigerator for >four (4) hours. 2. The blood has the potential for bacterial growth if allowed to infuse longer. 3. The blood components begin to break down after four (4) hours. 4. The blood will not be affected; this is a laboratory procedure.

2. The blood has the potential for bacterial growth if allowed to infuse longer.

83. The nurse is conducting a support group for clients diagnosed with Parkinson's disease and their significant others. Which information regarding psychosocial needs should be included in the discussion? 1. The client should discuss feelings about being placed on a ventilator. 2. The client may have rapid mood swings and become easily upset. 3. Pill-rolling tremors will become worse when the medication is wearing off. 4. The client may automatically start to repeat what another person says.

2. These are psychosocial manifestations of PD. These should be discussed in the support meeting.

A client comes to the emergency department complaining of a sudden onset of sharp, severe flank pain. During the physical examination, the client indicates that the pain, which comes in waves, travels to the suprapubic region. He states, "I can even feel the pain at the tip of my penis." Which of the following would the nurse suspect? 1- Acute glomerulonephritis 2- Ureteral stricture 3- Urinary calculi 4- Renal cell carcinoma

3

A client is admitted with increased ascites related to cirrhosis. Which nursing diagnosis should receive top priority? 1- Fatigue 2- Excess fluid volume 3- Ineffective breathing pattern 4- Imbalanced nutrition: Less than body requirements

3

A nurse who works in a clinic sees many patients with a variety of medical conditions. The nurse understands that a risk factor for UTIs is which of the following? 1- Hyperuricemia 2- Pancreatitis 3- Diabetes mellitus 4- Hyperparathyroidism

3

A woman comes to her health care provider's office with signs and symptoms of kidney stones. Which of the following should be the primary medical management goal? 1- Determine the stone type. 2- Relieve any obstruction. 3- Relieve the pain. 4- Prevent nephron destruction.

3

If medical treatments fail, which of the following invasive procedures is necessary for treating cardiomyopathy? 1. Cardiac catheterization 2. Coronary artery bypass graft (CABG) 3. Heart transplantation 4. Intra-aortic balloon pump (IABP)

3

On a visit to the clinic, a client reports the onset of early symptoms of rheumatoid arthritis. The nurse should conduct a focused assessment for: 1. Limited motion of joints. 2. Deformed joints of the hands. 3. Early morning stiffness. 4. Rheumatoid nodules.

3

The charge nurse is making assignments. Staffing includes a registered nurse with five (5) years of medical-surgical experience, a newly graduated registered nurse, and two (2) unlicensed assistive personnel (UAPs). Which client should be assigned to the most experienced nurse? 1. The 39-year-old client diagnosed with lower esophageal dysfunction who is complaining of pyrosis. 2. The 54-year-old client diagnosed with Barrett's esophagus who is scheduled to have an endoscopy this morning. 3. The 46-year-old client diagnosed with gastroesophageal reflux disease who has wheezes in all five (5) lobes. 4. The 68-year-old client who is three (3) days postoperative for hiatal hernia and needs to be ambulated four (4) times today.

3

The client diagnosed with Crohn's disease is crying and tells the nurse, "I can't take it anymore. I never know when I will get sick and end up here in the hospital." Which statement is the nurse's best response? 1. "I understand how frustrating this must be for you." 2. "You must keep thinking about the good things in your life." 3. "I can see you are very upset. I'll sit down and we can talk." 4. "Are you thinking about doing anything like committing suicide?"

3

The client diagnosed with ulcerative colitis is prescribed a low-residue diet. Which meal selection indicates the client understands the diet teaching? 1. Grilled hamburger on a wheat bun and fried potatoes. 2. A chicken salad sandwich and lettuce and tomato salad. 3. Roast pork, white rice, and plain custard. 4. Fried fish, whole grain pasta, and fruit salad.

3

The client is admitted to the medical unit with a diagnosis of acute diverticulitis. Which health-care provider's order should the nurse question? 1. Insert a nasogastric tube. 2. Start an IV with D5W at 125 mL/hr. 3. Put client on a clear liquid diet. 4. Place client on bedrest with bathroom privileges.

3

The client is diagnosed with an acute exacerbation of IBD. Which priority intervention should the nurse implement first? 1. Weigh the client daily and document in the client's chart. 2. Teach coping strategies such as dietary modifications. 3. Record the frequency, amount, and color of stools. 4. Monitor the client's oral fluid intake every shift.

3

The client is one (1) hour post-endoscopic retrograde cholangiopancreatogram (ERCP). Which intervention should the nurse include in the plan of care? 1. Instruct the client to cough forcefully. 2. Encourage early ambulation. 3. Assess for return of a gag reflex. 4. Administer held medications.

3

The clinic nurse is teaching a young woman about preventing recurrent urinary tract infections. What information should the nurse include? 1- Bathe daily. 2- Avoid voiding immediately after sexual intercourse. 3- Drink liberal amounts of fluids. 4- Void every 6 to 8 hours.

3

The nurse advises the patient with chronic pyelonephritis that he should: 1- Limit his fluid intake to 1.5 L/day to minimize bladder fullness, which could cause backward pressure on the kidneys. 2- Decrease his sodium intake to prevent fluid retention. 3- Increase fluids to 3 to 4 L/24 hours to dilute the urine. 4- Decrease his intake of calcium rich foods to prevent kidney stones.

3

The nurse has received the a.m. shift report. Which client should the nurse assess first? 1. The 44-year-old client diagnosed with peptic ulcer disease who is complaining of acute epigastric pain. 2. The 74-year-old client diagnosed with acute gastroenteritis who has had four (4) diarrhea stools during the night. 3. The 65-year-old client diagnosed with IBD who has tented skin turgor and dry mucous membranes. 4. The 15-year-old client diagnosed with food poisoning who has vomited several times during the night shift.

3

The nurse is assisting in the transport of a client with an indwelling catheter to the diagnostic studies unit. Which action made by the nursing assistant would require instruction? 1- The nursing assistant keeps the catheter and drainage bag together when moving the client. 2- The nursing assistant places the drainage bag on the client's abdomen for transport. 3- The nursing assistant places the drainage bag on the lower area of the wheelchair for transport. 4- The nursing assistant holds the drainage bag while the client moves to the wheelchair.

3

The nurse is caring for the immediate postoperative client who had a laparoscopic cholecystectomy. Which task could the nurse delegate to the unlicensed assistive personnel (UAP)? 1. Check the abdominal dressings for bleeding. 2. Increase the IV fluid if the blood pressure is low. 3. Ambulate the client to the bathroom. 4. Auscultate the breath sounds in all lobes.

3

The nurse is discussing the therapeutic diet for the client diagnosed with diverticulosis. Which meal indicates the client understands the discharge teaching? 1. Fried fish, mashed potatoes, and iced tea. 2. Ham sandwich, applesauce, and whole milk. 3. Chicken salad on whole-wheat bread and water. 4. Lettuce, tomato, and cucumber salad and coffee.

3

The nurse is planning the care of a client diagnosed with lower esophageal sphincter dysfunction. Which dietary modifications should be included in the plan of care? 1. Allow any of the client's favorite foods as long as the amount is limited. 2. Have the client perform eructation exercises several times a day. 3. Eat four (4) to six (6) small meals a day and limit fluids during mealtimes. 4. Encourage the client to consume a glass of red wine with one (1) meal a day.

3

The nurse, a licensed practical nurse (LPN), and an unlicensed assistive personnel (UAP) are caring for clients on a medical floor. Which nursing task would be most appropriate to assign to the LPN? 1. Assist the UAP to learn to perform blood glucose checks. 2. Monitor the potassium levels of a client with diarrhea. 3. Administer a bulk laxative to a client diagnosed with constipation. 4. Assess the abdomen of a client who has had complaints of pain.

3

The client diagnosed with Guillain-Barré syndrome is having difficulty breathing and is placed on a ventilator. Which situation warrants immediate intervention by the nurse? 1. The ventilator rate is set at 14 breaths per minute. 2. A manual resuscitation bag is at the client's bedside. 3. The client's pulse oximeter reading is 85%. 4. The ABG results are pH 7.40, PaO288, PaCO235, and HCO324.

3 A pulse oximeter reading of less than 93% warrants immediate intervention;a 90% peripheral oxygen saturation indicates a PaO2 of about 60 (normal,80 to 100). When the client is placed on the ventilator, this should cause the client's oxygen level to improve. 1. The rate of ventilation is usually 12 to 15 breaths per minute in adults who are on ventilators, so this rate does not require immediate intervention. 2. A manual resuscitation (Ambu) bag must be at the client's bedside in case the ventilator malfunctions; the nurse must bag the client. 4. These ABGs are within normal limits and do not warrant immediate intervention

The nurse is caring for a client diagnosed with hemorrhaging duodenal ulcer. Which collaborative interventions should the nurse implement? Select all that apply. 1. Perform a complete pain assessment. 2. Assess the client's vital signs frequently. 3. Administer a proton pump inhibitor intravenously. 4. Obtain permission and administer blood products. 5. Monitor the intake of a soft, bland diet.

3, 4

84. The nurse is caring for clients on a medical-surgical floor. Which client should be assessed first? 1. The 65-year-old client diagnosed with seizures who is complaining of a headache that is a "2" on a 1-to-10 scale. 2. The 24-year-old client diagnosed with a T10 spinal cord injury who cannot move his toes. 3. The 58-year-old client diagnosed with Parkinson's disease who is crying and worried about her facial appearance. 4. The 62-year-old client diagnosed with a cerebrovascular accident who has a resolving left hemiparesis.

3. Body image is a concern for clients diagnosed with PD. This client is the one client who is not experiencing expected sequelae of the disease.

74. The client diagnosed with PD is being discharged on carbidopa/levodopa (Sinemet), an antiparkinsonian drug. Which statement is the scientific rationale for combining these medications? 1. There will be fewer side effects with this combination than with carbidopa alone. 2. Dopamine D requires the presence of both of these medications to work. 3. Carbidopa makes more levodopa available to the brain. 4. Carbidopa crosses the blood-brain barrier to treat Parkinson's disease.

3. Carbidopa enhances the effects of levodopa by inhibiting decarboxylase in the periphery, thereby making more levodopa available to the central nervous system. Sinemet is the most effective treatment for PD.

Which sign/symptom will the nurse expect to assess in the client diagnosed with a vaso-occlusive sickle cell crisis? 1. Lordosis. 2. Epistaxis. 3. Hematuria. 4. Petechiae.

3. Hematuria.

The client diagnosed with sickle cell anemia is experiencing a vaso-occlusive sickle cell crisis secondary to an infection. Which medical treatment should the nurse anticipate the HCP ordering for the client? 1. Administer meperidine (Demerol) intravenously. 2. Admit the client to a private room and keep in reverse isolation. 3. Infuse D5W 0.33% NS at 150 mL/hr via pump. 4. Insert a 22-French Foley catheter with a urimeter.

3. Infuse D5W 0.33% NS at 150 mL/hr via pump.

82. Which is a common cognitive problem associated with Parkinson's disease? 1. Emotional lability. 2. Depression. 3. Memory deficits. 4. Paranoia.

3. Memory deficits are cognitive impairments. The client may also develop a dementia.

80. The client diagnosed with Parkinson's disease is being discharged. Which statement made by the significant other indicates an understanding of the discharge instructions? 1. "All of my spouse's emotions will slow down now just like his body movements." 2. "My spouse may experience hallucinations until the medication starts working." 3. "I will schedule appointments late in the morning after his morning bath." 4. "It is fine if we don't follow a strict medication schedule on weekends."

3. Scheduling appointments late in the morning gives the client a chance to complete ADLs without pressure and allows the medications time to give the best benefits.

The charge nurse is making assignments on a medical floor. Which client should be assigned to the most experienced nurse? 1. The client diagnosed with iron-deficiency anemia who is prescribed iron supplements. 2. The client diagnosed with pernicious anemia who is receiving vitamin B12 intramuscularly. 3. The client diagnosed with aplastic anemia who has developed pancytopenia. 4. The client diagnosed with renal disease who has a deficiency of erythropoietin

3. The client diagnosed with aplastic anemia who has developed pancytopenia.

The nurse is caring for clients on a medical floor. After the shift report, which client should be assessed first? 1. The client who is two thirds of the way through a blood transfusion and has had no complaints of dyspnea or hives. 2. The client diagnosed with leukemia who has a hematocrit of 18% and petechiae covering the body. 3. The client with peptic ulcer disease who called over the intercom to say that he is vomiting blood. 4. The client diagnosed with Crohn's disease who is complaining of perineal discomfort.

3. The client with peptic ulcer disease who called over the intercom to say that he is vomiting blood.

The nurse is working in a blood bank facility procuring units of blood from donors. Which client would not be a candidate to donate blood? 1. The client who had wisdom teeth removed a week ago. 2. The nursing student who received a measles immunization two (2) months ago. 3. The mother with a six (6)-week-old newborn. 4. The client who developed an allergy to aspirin in childhood.

3. The mother with a six (6)-week-old newborn.

The client diagnosed with iron-deficiency anemia is prescribed ferrous gluconate orally. Which should the nurse teach the client? 1. Take Imodium, an antidiarrheal, OTC for diarrhea. 2. Limit exercise for several weeks until a tolerance is achieved. 3. The stools may be very dark, and this can mask blood. 4. Eat only red meats and organ meats for protein.

3. The stools may be very dark, and this can mask blood.

A client with rheumatoid arthritis tells the nurse, "I know it is important to exercise my joints so that I won't lose mobility, but my joints are so stiff and painful that exercising is difficult." Which of the following responses by the nurse would be most appropriate? 1. "You are probably exercising too much. Decrease your exercise to every other day." 2. "Tell the physician about your symptoms. Maybe your analgesic medication can be increased." 3. "Stiffness and pain are part of the disease. Learn to cope by focusing on activities you enjoy." 4. "Take a warm tub bath or shower before exercising. This may help with your discomfort."

4

A female client who suffers from urethral strictures undergoes a dilation procedure. Following the procedure, she experiences a burning sensation while voiding. Which of the following instructions would be most helpful? 1- Encourage her to visit a local ostomy support group. 2- Advise her to cleanse her perineum frequently. 3- Urge her to apply moisture sealants. 4- Instruct her to take warm sitz baths.

4

Behavioral interventions for urinary incontinence can be coordinated by a nurse. A comprehensive program that incorporates timed voiding and urinary urge inhibition is referred to as what? 1- Voiding at given intervals 2- Prompted voiding 3- Interval voiding 4- Bladder retraining

4

If an indwelling catheter is necessary, which nursing intervention should be implemented to prevent infection? 1- Use a clean technique during insertion 2- Use a sterile technique to disconnect the catheter from the tubing to obtain urine specimens 3- Place the catheter bag on the client's abdomen when moving the client 4- Perform meticulous perineal care daily with soap and water

4

If an indwelling catheter is necessary, which nursing intervention should be implemented to prevent infection? 1- Use clean technique during insertion 2- Use sterile technique to disconnect the catheter from the tubing to obtain urine specimens 3- Place the catheter bag on the client's abdomen when moving the client 4- Perform meticulous perineal care daily with soap and water

4

The client diagnosed with diverticulitis is complaining of severe pain in the left lower quadrant and has an oral temperature of 100.6˚F. Which intervention should the nurse implement first? 1. Notify the health-care provider. 2. Document the findings in the chart. 3. Administer an oral antipyretic. 4. Assess the client's abdomen.

4

The client is placed on percutaneous endoscopic gastrostomy (PEG) tube feedings. Which occurrence warrants immediate intervention by the nurse? 1. The client tolerates the feedings being infused at 50 mL/hr. 2. The client pulls the nasogastric feeding tube out. 3. The client complains of being thirsty. 4. The client has green, watery stool.

4

The client is two (2) hours post-colonoscopy. Which assessment data warrant intermediate intervention by the nurse? 1. The client has a soft, nontender abdomen. 2. The client has a loose, watery stool. 3. The client has hyperactive bowel sounds. 4. The client's pulse is 104 and BP is 98/60.

4

The nurse is administering morning medications at 0730. Which medication should have priority? 1. A proton pump inhibitor. 2. A nonnarcotic analgesic. 3. A histamine receptor antagonist. 4. A mucosal barrier agent.

4

The nurse is caring for a client diagnosed with GERD. Which nursing interventions should be implemented? 1. Place the client prone in bed and administer nonsteroidal anti-inflammatory medications. 2. Have the client remain upright at all times and walk for 30 minutes three (3) times a week. 3. Instruct the client to maintain a right lateral side-lying position and take antacids before meals. 4. Elevate the head of the bed 30 degrees and discuss lifestyle modifications with the client.

4

The nurse is caring for a postoperative client who has a Kock pouch. Nursing assessment findings reveal abdominal pain, absence of bowel sounds, fever, tachycardia, and tachypnea. The nurse suspects which of the following? 1- Stoma ischemia 2- Postoperative pneumonia 3- Stoma retraction 4- Peritonitis

4

The nurse is caring for several older clients. For which client would the nurse be especially alert for signs and symptoms of pyelonephritis? 1- A client with acute renal failure 2- A client with a urinary tumor 3- A female client with preexisting chronic glomerulonephritis 4- A client with urinary obstruction

4

The nurse is preparing a client diagnosed with GERD for discharge following an esophagogastroduodenoscopy. Which statement indicates the client understands the discharge instructions? 1. "I should not eat for at least one (1) day following this procedure." 2. "I can lie down whenever I want after a meal. It won't make a difference." 3. "The stomach contents won't bother my esophagus but will make me nauseous." 4. "I should avoid orange juice and eating tomatoes until my esophagus heals."

4

The nurse is teaching the client diagnosed with diverticulosis. Which instruction should the nurse include in the teaching session? 1. Discuss the importance of drinking 1,000 mL of water daily. 2. Instruct the client to exercise at least three (3) times a week. 3. Teach the client about a eating a low-residue diet. 4. Explain the need to have daily bowel movements.

4

Which assessment data should the nurse assess in the client diagnosed with Guillain-Barré syndrome? 1. An exaggerated startle reflex and memory changes. 2. Cogwheel rigidity and inability to initiate voluntary movement. 3. Sudden severe unilateral facial pain and inability to chew. 4. Progressive ascending paralysis of the lower extremities and numbness.

4 Ascending paralysis is the classic symptom of Guillain-Barré syndrome 1. These signs/symptoms, along with sleep disturbances and nervousness, support the diagnosis of Creutzfeldt-Jakob disease. 2. These signs/symptoms support the diagnosis of Parkinson's disease. 3. These are signs/symptoms of trigeminal neuralgia.

The client admitted with rule-out Guillain-Barré syndrome has just had a lumbar puncture. Which intervention should the nurse implement post procedure? 1. Monitor the client for hypotension. 2. Apply pressure to the puncture site. . Test the client's cerebrospinal fluid. 4. Increase the client's fluid intake.

4 Increased fluid intake will help prevent a postprocedure headache, which may occur after a lumbar puncture. 1. Very little cerebrospinal fluid is removed the client. Therefore, hypotension is not a potential complication of this procedure. 2. A bandage is placed over the puncture site,and pressure does not need to be applied to the site. 3. The laboratory staff, not the nurse, complete tests on the cerebrospinal fluid; the nurse could label the specimens and take them to the laboratory

Which priority client problem should be included in the care plan for the client diagnosed with Guillain-Barré syndrome? 1. High risk for injury. 2. Fear and anxiety. 3. Altered nutrition. 4. Ineffective breathing pattern.

4 Guillain-Barré syndrome has ascendingparalysis causing respiratory failure.Therefore, breathing pattern is priority. 1. Safety is an important issue for the client,but this is not the priority client problem. 2. The client's psychological needs are important, but psychosocial problems are not priority over physiological problems. 3. Clients with this syndrome may have choking episodes and are at risk for inability to swallow as a result of the disease process, but this is not the priority nursing problem because weight loss is not an expected complication of this syndrome.

The client diagnosed with Guillain-Barré syndrome is on a ventilator. When the wife comes to visit she starts crying uncontrollably, and the client starts fighting the ventilator because his wife is upset. Which action should the nurse implement? 1. Tell the wife she must stop crying. 2. Escort the wife out of the room. 3. Medicate the client immediately. 4. Acknowledge the wife's fears.

4 It is scary for a wife to see her loved one with a tube down his mouth and all the machines around them. The nurse should help the wife by acknowledging her fears 1. This action does not address the wife's fears, and telling her to stop crying will not help the situation. 2. Making the wife leave the room will further upset the client and the client's wife. 3. Medicating the client will not help the wife, but if the nurse can calm the wife,then it is hoped the client will calm down.

The student nurse asks the nurse, "What is sickle cell anemia?" Which statement by the nurse would be the best answer to the student's question? 1. "There is some written material at the desk that will explain the disease." 2. "It is a congenital disease of the blood in which the blood does not clot." 3. "The client has decreased synovial fluid that causes joint pain." 4. "The blood becomes thick when the client is deprived of oxygen."

4. "The blood becomes thick when the client is deprived of oxygen."

The nurse and an unlicensed assistive personnel (UAP) are caring for clients on an oncology floor. Which nursing task would be delegated to the UAP? 1. Assess the urine output on a client who has had a blood transfusion reaction. 2. Take the first 15 minutes of vital signs on a client receiving a unit of PRBCs. 3. Auscultate the lung sounds of a client prior to a transfusion. 4. Assist a client who received 10 units of platelets in brushing the teeth.

4. Assist a client who received 10 units of platelets in brushing the teeth.

The male client with sickle cell anemia comes to the emergency room with a temperature of 101.4˚F and tells the nurse that he is having a sickle cell crisis. Which diagnostic test should the nurse anticipate the emergency room doctor ordering for the client? 1. Spinal tap. 2. Hemoglobin electrophoresis. 3. Sickle-turbidity test (Sickledex). 4. Blood cultures.

4. Blood cultures.

79. The nurse researcher is working with clients diagnosed with Parkinson's disease. Which is an example of an experimental therapy? 1. Sterotactic pallidotomy/thalamotomy. 2. Dopamine receptor agonist medication. 3. Physical therapy for muscle strengthening. 4. Fetal tissue transplantation.

4. Fetal tissue transplantation has shown some success in PD, but it is an experimental and highly controversial procedure.

81. The nurse is admitting a client with the diagnosis of Parkinson's disease. Which assessment data support this diagnosis? 1. Crackles in the upper lung fields and jugular vein distention. 2. Muscle weakness in the upper extremities and ptosis. 3. Exaggerated arm swinging and scanning speech. 4. Masklike facies and a shuffling gait.

4. Masklike facies and a shuffling gait are two clinical manifestations of PD.

The client receiving a unit of PRBCs begins to chill and develops hives. Which action should be the nurse's first response? 1. Notify the laboratory and health-care provider. 2. Administer the histamine-1 blocker, Benadryl, IV. 3. Assess the client for further complications. 4. Stop the transfusion and change the tubing at the hub.

4. Stop the transfusion and change the tubing at the hub.

75. The nurse caring for a client diagnosed with Parkinson's disease writes a problem of "impaired nutrition." Which nursing intervention would be included in the plan of care? 1. Consult the occupational therapist for adaptive appliances for eating. 2. Request a low-fat, low-sodium diet from the dietary department. 3. Provide three (3) meals per day that include nuts and whole-grain breads. 4. Offer six (6) meals per day with a soft consistency.

4. The client's energy levels will not sustain eating for long periods. Offering frequent and easy-to-chew (soft) meals of small proportions is the preferred dietary plan.

78. The nurse is planning the care for a client diagnosed with Parkinson's disease. Which would be a therapeutic goal of treatment for the disease process? 1. The client will experience periods of akinesia throughout the day. 2. The client will take the prescribed medications correctly. 3. The client will be able to enjoy a family outing with the spouse. 4. The client will be able to carry out activities of daily living.

4. The major goal of treating PD is to maintain the ability to function. Clients diagnosed with PD experience slow, jerky movements and have difficulty performing routine daily tasks.

A client with diabetes comes to the clinic for a follow-up visit. The nurse reviews the client's glycosylated hemoglobin test results. Which result would indicate to the nurse that the client's blood glucose level has been well-controlled? 8.5% 7.5 % 8.0% 6.5%

6.5%

A health care provider prescribes short-acting insulin for a patient, instructing the patient to take the insulin 20 to 30 minutes before a meal. The nurse explains to the patient that Humulin-R, taken at 6:30 AM will reach peak effectiveness by: 10:30 AM. 2:30 PM. 8:30 AM. 12:30 PM.

8:30 AM.

19. An outpatient who has developed heart failure after having an acute myocardial infarction has a new prescription for carvedilol (Coreg). After 2 weeks, the patient returns to the clinic. The assessment finding that will be of most concern to the nurse is that the patient a. has BP of 88/42. b. has an apical pulse rate of 56 .c. complains of feeling tired. d. has 2+ pedal edema.

A

During an assessment of a patient's abdomen, a pulsating abdominal mass is noted by the healthcare provider. Which of the following should be the healthcare provider's next action? A. Assess femoral pulses B. Obtain a bladder scan C. Measure the abdominal circumference D. Ask the patient to perform a Valsalva maneuver

A

The client w/ MG is prescribed the cholinesterase inhibitor neostigmine (Prostigmin). Which data indicate the med is effective? A) Client is able to feed self independently B) The client is able to blink the eyes w/out tearing C) The client denies and nasue/ vomit when eating D_ The client denies any pain when performing ROM

A

An older adult resident in a long-term-care facility becomes confused and agitated, telling the nurse, "Get out of here! You're going to kill me!" Which action will the nurse take first? A. Check the resident's oxygen saturation. B. Do a complete neurologic assessment. C. Give the prescribed PRN lorazepam (Ativan). D. Notify the resident's primary care provider

A A common reason for sudden confusion in older clients is hypoxemia caused by undiagnosed pneumonia. The nurse's first action should be to assess oxygenation by checking the pulse oximetry. Determining the cause of the confusion is the primary goal of the RN.

The nurse is caring for a client with peripheral arterial disease (PAD). For which symptoms does the nurse assess? A. Reproducible leg pain with exercise B. Unilateral swelling of affected leg C. Decreased pain when legs are elevated D. Pulse oximetry reading of 90%

A Claudication (leg pain with ambulation due to ischemia) is reproducible in similar circumstances. Unilateral swelling is typical of venous problems such as deep vein thrombosis. With PAD, pain decreases with legs in the dependent position. Pulse oximetry readings reflect the amount of oxygen bound to hemoglobin; PAD results from atherosclerotic occlusion of peripheral arteries.

A pt is taking isoniazid, rifampin, pyrazinamide, and ethambutol for TB. The pt calls to report visual changes, including blurred vision and reduced visual fields. Which medication may be causing these changes? A. Ethambutol B. Isoniazid C. Pyrazinamide D. Rifampin

A Ethambutol can cause optic neuritis, leading to blindness at high doses. When discovered early and the drug is stopped, problems can usually be reversed. Both isoniazid and pyrazinamide may cause liver failure; side effects of major concern include jaundice, bleeding, and abdominal pain. Rifampin will cause the urine and all other secretions to have a yellowish-orange color; this is harmless. Contact lenses will also be stained and oral contraceptives will be less effective.

An elderly client with pneumonia may appear with which of the following symptoms first? A. Altered mental status and dehydration B. Fever and chills C. Hemoptysis and dyspnea D. Pleuritic chest pain and cough

A Fever, chills, hemoptysis, dyspnea, cough, and pleuritic chest pain are common symptoms of pneumonia, but elderly clients may first appear with only an altered mental status and dehydration due to a blunted immune response.

A nurse is teaching a client with TB about dietary elements that should be increased in the diet. The nurse suggests that the client increase intake of: A. Meats and citrus fruits B. Grains and broccoli C. Eggs and spinach D. Potatoes and fish

A Needs to increase intake of protein, iron, and vitamin C

A client has just undergone arterial revascularization. Which statement by the client indicates a need for further teaching related to postoperative care? A. "My leg might turn very white after the surgery." B. "I should be concerned if my foot turns blue." C. "I should report a fever or any drainage." D. "Warmness, redness, and swelling are expected."

A Pallor is one of the signs of decreased perfusion along with increased pain, poikilothermia, paresthesia, pulselessness, and paralysis

A client was infected with TB 10 years ago but never developed the disease. He's now being treated for cancer. The client begins to develop signs of TB. This is known as which of the following types of infection? A. Active infection B. Primary infection C. Superinfection D. Tertiary infection

A Some people carry dormant TB infections that may develop into active disease. In addition, primary sites of infection containing TB bacilli may remain inactive for years and then activate when the client's resistance is lowered, as when a client is being treated for cancer. There's no such thing as tertiary infection, and superinfection doesn't apply in this case.

The community health nurse is planning tuberculosis treatment for a client who is homeless and heroin-addicted. Which action will be most effective in ensuring that the client completes treatment? A. Arrange for a health care worker to watch the client take the medication. B. Give the client written instructions about how to take prescribed medications. C. Have the client repeat medication names and side effects. D. Instruct the client about the possible consequences of nonadherence.

A The client is unlikely to adhere to long-term treatment unless med administration is directly supervised. The best option is to arrange for DOT.

The nurse is assigned to all of these clients. Which client should be assessed first? A. The client who had percutaneous transluminal angioplasty (PTA) of the right femoral artery 30 minutes ago B. The client admitted with hypertensive crisis who has a nitroprusside (Nipride) drip and blood pressure of 149/80 mm Hg C. The client with peripheral vascular disease who has a left leg ulcer draining purulent yellow fluid D. The client who had a right femoral-popliteal bypass 3 days ago and has ongoing edema of the foot

A The client who had PTA should have checks of vascular status and vital signs every 15 minutes in the first hour after the procedure.

When auscultating the chest of a client with pneumonia, the nurse would expect to hear which of the following sounds over areas of consolidation? A. Bronchial B. Bronchovesicular C. Tubular D. Vesicular

A Bronchial Chest auscultation reveals bronchial breath sounds over areas of consolidation. Bronchovesicular are normal over midlobe lung regions, tubular sounds are commonly heard over large airways, and vesicular breath sounds are commonly heard in the bases of the lung fields.

A nurse is auscultating the lower lung fields of a pneumonia pt. She hears coarse crackles and ids the problem as impaired oxygenation. She knows the underlying physiologic condition associated with pt condition is A. hypoxemia B. hyperemia C. hypocapnia D. hypercapnia

A hypoxemia

Which test results are indicative of active TB? A. induration of 11mm and positive sputum B. sputum tests positive for blood C. positive chest x-ray for TB D. positive chest x-ray and clinical symptoms

A induration 11mm and positive sputum

A nurse is caring for an 89-year-old client admitted with pneumonia. He has an IV of normal saline running at 100 mL/hr and antibiotics that were initiated in the emergency department 3 hours ago. He has oxygen at 2 liters/nasal cannula. What assessment finding by the nurse indicates that goals for a priority diagnosis have been met for this client? A) The client is alert and oriented to person, place, and time. B) Blood pressure is within normal limits and client's baseline. C) Skin behind the ears demonstrates no redness or irritation. D) Urine output has been >30 mL/hr per Foley catheter

A) The client is alert and oriented to person, place, and time.

What is the rationale for using CPAP to treat sleep apnea? A) positive air pressure holds the airway open B) negative air pressure holds the airway closed C) delivery of oxygen facilitates respiratory effort D) alternating waves of air stimulate breathing

A) positive air pressure holds the airway open

A pt recently released from prison has just tested positive for TB. What teaching points does the community health nurse want to stress for this pt about the meds? Select All That Apply A. Not taking the meds could lead to an infection that is difficult to treat or to total drug resistance. B. The meds may cause nausea, so take them at bedtime. C. The pt is not contagious after 2 to 3 consecutive weeks of treatment. D. These meds have to be taken for 2 years.

A, B, C Not taking the meds as prescribed can lead to an infection that is difficult to treat or to total drug resistance. The meds may cause nausea and are best taken at bedtime to prevent this. The pt is generally not contagious after 2 to 3 weeks of consecutive treatment BUT must show improvement in the condition. The combination regimen for treatment of TB has decreased treatment time from 6 to 12 months to 6 months. TB meds can cause liver failure, but not kidney failure.

a client with peptic ulcer reports epigastric pain that frequently causes the client to wake up during the night. the nurse should instruct the client to do which activities? select all that apply. a. obtain adequate rest to reduce stimulation b. eat small, frequent meals throughout the day c. take all medications on time as prescribed d. sit up for 1 hour when awakened at nigh e. stay away from crowded areas

A, B, C, D

The healthcare provider is teaching a group of senior citizens about risk factors for heart failure. Which of these factors will the healthcare provider include in the teaching?Select all that apply. A) High sodium intake B) History of preeclampsia C) Obesity D) Sleep apnea E) Increased high density lipoproteins (HDL) F) Hypertension

A, B, C, D, F

The nurse educates a primary HTN patient on lifestyle changes. Which ones should be included in her teaching? A. consume more fruits/veggies B. Monitor/lose weight C. Limit alchoholic drinks to 3 per day or less D. Regular exercise (walking) E. Limit sodium intake to 3200 mg per day

A, B, D

Which are risk factors that are known to contribute to atherosclerosis-related diseases? (Select all that apply.) A. Low-density lipoprotein cholesterol (LDL-C) of 160 mg/dL B. Smoking C. Aspirin (acetylsalicylic acid [ASA]) consumption D. Type 2 diabetes E. Vegetarian diet

A, B, D Having an LDL-C value of less than 100 mg/dL is optimal; 100 to 129 mg/dL is near or less than optimal; with LDL-C 130 to 159 mg/dL (borderline high), the client is advised to modify diet and exercise. Smoking is a modifiable risk factor and should be avoided or terminated, and diabetes is a risk factor for atherosclerotic disease.

The nurse is caring for a pt who often coughs and chokes while eating and taking his meds, but the pt insists he is fine. The nurse recognizes this as a priority pt problem of risk for aspiration. The nursing interventions she would implement to prevent aspiration pneumonia are Select All That Apply A. HOB always elevated during meals B. Monitor pt ability to swallow small bites C. Give small frequent drinks of thin liquid D. Consult a nutritionist and obtain swallow studies E. Monitor the patient's ability to swallow saliva F. Place pt on NPO status til swallowing returns to normal

A, B, D, E

A pt with TB is homeless and been living in shelters for the past 7 months, he asks the nurse why he must take so many meds. What information will the nurse provide in answering this question? Select All That Apply A. Combination drug therapy is effective in preventing transmission. B. Combination drug therapy is the most effective method of treating TB. C. Combination drug therapy will decrease the length of required treatment to 2 months. D. Multiple drug regimens destroy organisms as quickly as possible. E. The use of multiple drugs reduces the emergence of drug-resistant organisms.

A, B, D, E Combination drug therapy is the most effective method for treating TB and preventing transmission. Multiple drug regimens can destroy organisms as quickly as possible and reduce the emergence of drug-resistant organisms. Combination drug therapy will decrease the required length of time for treatment, the length of treatment is decreased to 6 months from 6 to 12 months.

The nurse in the cardiology clinic is reviewing teaching about hypertension, provided at the client's last appointment. Which actions by the client indicate that teaching has been effective? (Select all that apply.) A. Has maintained a low-sodium, no-added-salt diet B. Has lost 3 pounds since last seen in the clinic C. Cooks food in palm oil to save money D. Exercises once weekly E. Has cut down on caffeine

A, B, E Clients with hypertension should consume low-sodium foods and should avoid adding salt. Weight loss can result in lower blood pressure. Caffeine promotes vasoconstriction, thereby elevating blood pressure.

A nurse knows that a patient may have pyelonephritis if they present with: A. Fever/chills B. Increased WBC in urine C. Diarrhea D. Unilateral pitting edema E. CVA Tenderness F. Blurry Vision G. Gynecomastia H. Cloudy, blood, foul smelling urine

A, B, E, H

Which conditions does the nurse recognize as a risk for developing aspiration pneumonia? Select All That Apply A. continuous tube feed B. bronchoscopy procedure C. MRI D. decreased LOC E. stroke F. chest tube

A, B. D. E continuous tube feed bronchoscopy procedure decreased LOC stroke

A patient presenting to the ER with a hypertensive crisis (BP greater than 180/120), may have damage to which of the following? A. Brain B. Kidney C. Liver D. Heart E. Stomach F. Eyes

A, B. D. F CVA retinopathy heart failure renal failure IV beta blocker will be ordered immediately for a pt in a hypertensive crisis

The patient understand that which of the following are factors that he can change to decrease his risk of HTN? Select All That Apply A. smoking B. family history C. Alcohol consumption D. increased LDL E. Sedentary lifestyle

A, C. D, E Pt can change all but his family history

The silent killer, essential HTN, sometimes doesn't have obvious s/s, but some that may be reported by the patient include: Select All That Apply A. Dizziness B. Kidney disease C. Headache D. Syncope (fainting) E. Hot/flushed F. Nose bleed (epistaxis) G. Diabetes

A, C. D, E, F Kidney disease and diabetes are not s/s and are a factor in secondary HTN, not primary

A pt is seen in the HCP office and dx with community-acquired pneumonia. The nurse knows the most common symptoms that this pt may have is A. dyspnea B. abdominal pain C. back pain D. hypoxemia E. chest discomfort F. a smoker

A, D, E dyspnea hypoxemia chest discomfort

A pt is suspected on having community-acquired pneumonia. The nurse anticipates which of the following tests to be done to dx pt A. sputum gram stain B. Pulmonary function test C. fluorescein bronchoscopy D. peak flow meter measurement E. chest x-ay

A, E sputum gram stain CXR

The nurse recognizes that isoniazid, rifampin and pyrazinamide can cause impairment of the liver. She will look for the following s/s of liver impairment: A. dark urine B. weight gain C. diarrhea D. tremors E. yellowing of skin, eyes , and/or hard palate

A,, E will have anorexia not wt gain diarrhea and tremors are not s/s of liver issues pt wil have clay-colored stools

The client diagnosed w/ MG is admitted w/ an acute exacerbation. Which interventions should the nurse implement? SELECT ALL APPLY. A) Assist the pt. to turn and cough every 2 hrs B) Place client in high/ semi fowlers C) assess client's pulse ox reading every shift D) Plan meals to promote medication effectiveness E) Monitor client's serum anticholinesterase levels

A,B,D

A patient is taking an antacid and H2 receptor antagonist to treat their PUD. What statement indicates the patient needs further teaching? A. "I can take both medications at the same time." B. "My antacid neutralizes my stomach acid." C. "My H2 receptor antagonist inhibits stomach acid secretion." D. "I should take these medications an hour apart."

A. "I can take both medications at the same time." -They should be taken 1 hour apart

An active 45-year-old schoolteacher with COPD is taking prednisone asks if it is necessary to get a flu shot. What is the best response by the nurse? A. "Yes, flu shots are highly recommended for patients with chronic illness and/or patients who are receiving immunotherapy." B. "No, flu shots are only recommended for patients 50 years old and older." C. "Yes, it will help minimize the risk of triggering of exacerbation of COPD." D. "No, patients who are active, not living in a nursing home, and not health care providers do not need the flu shot."

A. "Yes, flu shots are highly recommended for patients with chronic illness and/or patients who are receiving immunotherapy."

The nurse is providing discharge instructions about pneumonia to a patient and family. Which discharge information must the nurse be sure to include? A. Complete antibiotics as prescribed, rest, drink fluids, and minimize contact with crowds. B. Take all antibiotics as ordered, resume diet and all activities as before hospitalization. C. No restrictions regarding activities, diet and rest because the patient is fully recovered when discharged. D. Continue antibiotics only until no further signs of pneumonia are present; avoid exposing immunosuppressed individuals.

A. Complete antibiotics as prescribed, rest, drink fluids, and minimize contact with crowds.

What should the patient with Bell's palsy be cautioned against? A. Cornea dryness B. Driving while experiencing diplopia C. Sudden movement of the head when bending over D. Contamination from the affected eye to the other eye

A. Cornea drynessWith Bell's palsy, the eyelid on the affected side often does not close tightly. Eye drops are used during the daytime, and patches are worn at night. The other options are not related to Bell's palsy.

A nurse is performing a physical assessment of a client with ulcerative colitis. Which of the following symptoms is most often associated with a serious complication of this disorder? A. Decreased bowel sounds B. Loose, blood-tinged stools C. Distention of the abdomen D. Intense abdominal discomfort

A. Decreased bowel sounds Decreased intestinal motility is associated with serious problems, such as perforation or toxic megacolon. Loose, blood-tinged stools are an uncomfortable but less serious manifestation. Distention of the abdomen is an expected response that is not of primary concern at this time. Intense pain is a symptom of ulcerative colitis, not a complication.

A client has a history of gastroesophageal reflux disease (GERD). Why should the nurse also monitor the client for clinical manifestations of heart disease? A. Esophageal pain may imitate the symptoms of a heart attack. B. GERD may predispose to heart disease. C. Strenuous exercise may exacerbate reflux problems. D. Similar changes in laboratory studies may occur in both cardiac and reflux problems.

A. Esophageal pain may imitate the symptoms of a heart attack. Epigastric pain is common with GERD and may be present with unstable angina and myocardial infarction.

The nurse is caring for an older adult male client who reports stomach pain and heartburn. Which syndrome is most significant in determining whether the client's ulceration is gastric or duodenal in origin? A. Pain occurs 1 1/2 to 3 hours after a meal, usually at night. B. Pain is worsened by the ingestion of food. C. The client has a malnourished appearance. D. The client is a man older than 50 years.

A. Pain occurs 1 1/2 to 3 hours after a meal, usually at night. A key symptom characteristic of duodenal ulcers is that pain usually awakens the client between 1 AM and 2 AM, occurring 1 1/2 to 3 hours after a meal.

A client was diagnosed with ulcerative colitis. Two months after the diagnosis, the client is readmitted for an exacerbation of the illness. The client is weak, thin, and irritable. The client states, "I am now ready for surgery to create an ileostomy." Which nursing intervention will best meet the client's priority need? A. Replace the client's fluids and electrolytes B. Help the client gain weight C. Teach the client how to use the ileostomy appliance D. Encourage client interaction with other clients who have an ileostomy

A. Replace the client's fluids and electrolytes

The client is experiencing bleeding related to peptic ulcer disease (PUD). Which nursing intervention is the highest priority? A. Starting a large-bore intravenous (IV) B. Administering intravenous (IV) pain medication C. Preparing equipment for intubation D. Monitoring the client's anxiety level

A. Starting a large-bore intravenous (IV) A large-bore IV should be placed as requested, so that blood products can be administered.

During the first 15 minutes of the blood administration the nurse stays with the patient obtaining baseline vital signs and vital signs after 15 minutes. The nurse notes that the patient's temperature has increased from 37.0C to 38.2C, the blood pressure has decreased from 120/80 to 90/40, and the patient complains of itching, and shortness of breath. What is the nurse's priority action? A. Stop the packed red blood cells and start Normal saline to keep vein open B. Call the blood bank C. Give Tylenol D. Find the charge nurse to double check the primary nurse's findings

A. Stop the packed red blood cells and start Normal saline to keep vein open

The nurse is caring for a client diagnosed with rule out peptic ulcer disease. Which test confirms this diagnosis? A. Esophagogastroduodenoscopy B. Magnetic resonance imaging C. Occult blood test D. Gastric acid stimulation.

A. The esophagogastroduodenoscopy (EGD) is an invasive diagnostic test which visualizes the esophagus, stomach, and duodenum to accurately diagnose an ulcer and evaluate the effectiveness of the clients treatment.

For a patient who had a right hemisphere stroke the nurse establishes a nursing diagnosis of a. risk for injury related to denial of deficits and impulsiveness. b. impaired physical mobility related to right-sided hemiplegia. c. impaired verbal communication related to speech-language deficits. d. ineffective coping related to depression and distress about disability.

A. The patient with right-sided brain damage typically denies any deficits and has poor impulse control, leading to risk for injury when the patient attempts activities such as transferring from a bed to a chair. Right-sided brain damage causes left hemiplegia. Left-sided brain damage typically causes language deficits. Left-sided brain damage is associated with depression and distress about the disability.

Which of the following best describes a paracentesis? A. Transabdominal removal of fluid from the peritoneal cavity B. Removal of fluid from the pleural space C. An opening into the stomach from the abdominal wall, made surgically for the introduction of food

A. Transabdominal removal of fluid from the peritoneal cavity

which of the following dietary measures would be useful in preventing esophageal reflux? a. eating small, frequent meals b. increasing fluid intake c. avoiding air swallowing with meals d. adding a bedtime snack to the dietary plan

A. eating small, frequent meals

the client attends two sessions with the dietitian to learn about diet modifications to minimize gastroesophageal reflux. the teaching would be considered successful if the client decreases the intake of which of the following foods? a. fats b. high-sodium foods c. carbs d. high-calcium foods

A. fats

3. A nurse is reviewing the arterial blood gas results of several clients. Which client's arterial blood gas result indicates metabolic acidosis? A. pH: 7.32 PaCO2: 32 HCO3: 16 B. pH: 7.34 PaCO2: 50 HCO3: 27 C. pH: 7.46 PaCO2: 33 HCO3: 18 D. pH: 7.48 PaCO2: 48 HCO3: 29

A. pH: 7.32 PaCO2: 32 HCO3: 16

the nurse is caring for a client who has just had an upper GI endoscopy. the client's vital signs must be taken every 30 minutes for 2 hours after the procedure. the nurse assigns an unlicensed nursing personnel (UAP) to take the vital signs. one hour later, the UAP reports the client, who was previously afebrile, has developed a temperature of 101.8F. what should the nurse do in response to these reported assessment data? a. promptly assess the client for potential perforation b. tell the assistant to change thermometers and retake the temperature c. plan to give the client acetaminophen to lower the temperature d. ask the UAP to bathe the client with tepid water

A. promptly assess the client for potential perforation.

the client is scheduled to have an upper gastrointestinal tract series of x-rays. following the x-rays. the nurse should instruct the client to: a. take a laxative b. follow a clear liquid diet c. administer an enema d. take an antiemetic

A. take a laxative

A patient is admitted to the hospital with a left hemiplegia. To determine the size and location and to ascertain whether a stroke is ischemic or hemorrhagic, the nurse anticipates that the health care provider will request a a. CT scan b. lumbar puncture c. cerebral arteriogram d. positron emission tomography (PET)

A: CT scan- A CT scan is the most commonly used diagnostic test to determine the size and location of the lesion and to differentiate a thrombotic stroke from a hemorrhagic stroke. Positron emission tomography (PET) will show the metabolic activity of the brain and provide a depiction of the extent of tissue damage after a stroke. Lumbar punctures are not performed routinely because of the chance of increased intracranial pressure causing herniation. Cerebral arteriograms are invasive and may dislodge an embolism or cause further hemorrhage; they are performed only when no other test can provide the needed information.

During the acute phase of a stroke, the nurse assesses the patient's vital signs and neurologic status every 4 hours. A cardiovascular sign that the nurse would see as the body attempts to increase cerebral blood flow is a. hypertension b. fluid overload c. cardiac dysrhythmias d. S3 and S4 heart sounds

A: Hypertension- The body responds to the vasopasm and a decreased circulation to the brain that occurs with a stroke by increasing the BP, frequently resulting in hypertension. The other options are important cardiovascular factors to assess, but they do not result from impaired cerebral blood flow.

A newly admitted patient who has suffered a right sided brain stroke has a nursing diagnosis of disturbed visual sensory perception related to homonymous hemianopsia. Early in the care of the patient, the nurse should a. place objects on the right side within the patient's field of vision b. approach the patient from the left side to encourage the patient to turn the head c. place objects on the patient's left side to assess the patient's ability to compensate d. patch the affected eye to encourage the patient to turn the head to scan the environment

A: Place objects on the right side within the patient's field of vision- the presence of homonymous hemianopia in a patient with right-hemisphere brain damage causes a loss of vision in the left field. Early in the care of the patient, objects should be placed on the right side of the patient in the field of vision, and the nurse should approach the patient from the right side. Later in treatment, patients should be taught to turn the head and scan the environment and should be approached from the affected side to encourage head turning. Eye patches are used if patients have diplopia (double vision).

Four days following a stroke, a patient is to start oral fluids and feedings. Before feeding the patient, the nurse should first a. check the patient's gag reflex b. order a soft diet for the patient c. raise the head of the bed to sitting position d. evaluate the patient's ability to swallow small sips of ice water

A: check the patient's gag reflex- the first step in providing oral feedings for a patient with a stroke is ensuring that the patient has an intact gag reflex because oral feedings will not be provided if gag reflex is impaired. The nurse should then evaluate the patient's ability to swallow ice chips or ice water after placing the patient in an upright position

The nurse is planning the care of a client diagnosed with aplastic anemia. Which interventions should be taught to the client? Select all that apply. 1. Avoid alcohol. 2. Pace activities. 3. Stop smoking. 4. Eat a balanced diet. 5. Use a safety razor.

ANSWER: 1, 2, 41. Alcohol consumption interferes with the absorption of nutrients.2. The client will be short of breath with activity and therefore should pace activities.3. Although all clients should be told to stop smoking, smoking will not directly affect the client's diagnosis.4. The client should eat a well-balanced diet to be able to manufacture blood cells. 5. The client should use an electric razor to diminish the risk of cuts and bleeding.

A patient has obstructive sleep apnea (OSA). What should the nurse include in the teaching related to ways the patient can minimize the negative effects of OSA? Select all that apply.' ARaise the head end of the bed slightly. BUse an oral appliance that prevents the airway from collapsing. CSleep in a side-lying position. DAvoid consuming alcoholic beverages several hours before going to sleep. ETake sleeping pills before going to bed.

ARaise the head end of the bed slightly. BUse an oral appliance that prevents the airway from collapsing. CSleep in a side-lying position. DAvoid consuming alcoholic beverages several hours before going to sleep.

In patients with Obstructive Sleep Apnea who are prescribed to use a PAP device at home. What is the patient education topic that the nurse should prioritize? Daily exercise regimen. Nutrition Having a sleep log to count the number of hours of sleep Adherence to the use of PAP machine

Adherence to the use of PAP machine

A patient newly diagnosed with type 1 diabetes has an unusual increase in blood glucose from bedtime to morning. The physician suspects the patient is experiencing insulin waning. Based on this diagnosis, the nurse will expect which of the following changes to the patient's medication regimen? Administering a dose of intermediate-acting insulin before the evening meal Decreasing evening bedtime dose of intermediate-acting insulin and administering a bedtime snack Changing the time of injection of evening intermediate-acting insulin from dinnertime to bedtime Increasing morning dose of long-acting insulin

Administering a dose of intermediate-acting insulin before the evening meal

A nurse is caring for a diabetic patient with a diagnosis of nephropathy. What would the nurse expect the urinalysis report to indicate? Bacteria Red blood cells Albumin White blood cells

Albumin

14. A patient with a history of renal calculi is hospitalized with gross hematuria and severe colicky left flank pain that radiates to his left testicle. In planning care for the patient, the nurse gives the highest priority to the nursing diagnosis of a. acute pain related to irritation by the stone. b. deficient fluid volume related to inadequate intake. c. risk for infection related to urinary system damage. d. risk for nausea related to pain and renal colic.

Answer: A Rationale: Although all the diagnoses are appropriate, the initial nursing actions should focus on relief of the acute pain. Cognitive Level: Application Text Reference: p. 1173 Nursing Process: Diagnosis NCLEX: Physiological Integrity

25. After her bath, a 62-year-old patient asks the nurse for a perineal pad, saying that she uses them because sometimes she leaks urine when she laughs or coughs. Which intervention is most appropriate to include in the care plan for the patient? a. Teach the patient how to perform Kegel exercises. b. Demonstrate how to perform Credé's maneuver. c. Place commode at the patient's bedside. d. Assist the patient to the bathroom q3hr.

Answer: A Rationale: Exercises to strengthen the pelvic floor muscles will help reduce stress incontinence. The Credé maneuver is used to help empty the bladder for patients with overflow incontinence. Placing the commode close to the bedside and assisting the patient to the bathroom are helpful for functional incontinence. Cognitive Level: Application Text Reference: pp. 1181-1184 Nursing Process: Planning NCLEX: Health Promotion and Maintenance

3. The nurse determines that instruction regarding prevention of future UTIs for a patient with cystitis has been effective when the patient states, a. "I will empty my bladder every 3 to 4 hours during the day." b. "I can use vaginal sprays to reduce bacteria." c. "I will wash with soap and water before sexual intercourse." d. "I will drink a quart of water or other fluids every day."

Answer: A Rationale: Voiding every 3 to 4 hours is recommended to prevent UTIs. Use of vaginal sprays is discouraged. The bladder should be emptied before and after intercourse, but cleaning with soap and water is not necessary. A quart of fluids is insufficient to provide adequate urine output to decrease risk for UTI. Cognitive Level: Application Text Reference: p. 1161 Nursing Process: Evaluation NCLEX: Health Promotion and Maintenance

Which of the following symptoms you as the nurse expect to see in the patient with primary progressive multiple sclerosis? (Select All that Apply): A) Unilateral Vision Loss B) Fatigue C) Diarrhea D) Intention tremors E) Paralytic ileus

Answer: A, B and D.

The client is experiencing a myasthenic crisis. Which of the following is a priority action of the following ordered actions? A) Insert NG tube B) Administer Ativan C) Monitor I&O D) Immediately stop anticholinesterase medications

Answer: A. Inserting the NG tube is the priority because it will help reduce risk for aspiration. The patient experiencing a myasthenic crisis is at a large risk for respiratory failure due to dysphagia and extreme muscle weakness. All priority actions should be focused on respiratory assessment and support. Ativan and any other sedating medication should NEVER be administered. Stopping anticholinesterase medications is associated with a cholinergic crisis. Monitoring I&O is important, but not as important as NG tube

The nurse is teaching the client with MS about the use of corticosteroids for treatment. Which of the following statements, if made by the patient indicates correct understanding? A) I should watch for side effects such as euphoria and insomnia while taking this medication B) This medication will need to be administered for at least 2 weeks before I begin to see improvements in my condition C) The corticosteroids will reduce my chances of relapsing in the future D) I could see flu-like symptoms while taking this medication

Answer: A. Some side effects of corticosteroid use include euphoria, mood changes, and insomnia. This medication should only be used for short periods of time (3-5 days) and is often tapered off. This medication is for use in shortening the duration of a relapse, not preventing relapse. Flu-like medications are often seen in Interferon beta-1a or 1b medications (Betaseron, avonex) which are used to for long-term treatment of MS.

5. A 34-year-old patient with diabetes mellitus is hospitalized with fever, anorexia, and confusion. The health care provider suspects acute pyelonephritis when the urinalysis reveals bacteriuria. An appropriate collaborative problem identified by the nurse for the patient is potential complication a. hydronephrosis. b. urosepsis. c. acute renal failure. d. chronic pyelonephritis.

Answer: B Rationale: Infection can easily spread from the kidney to the circulation, causing urosepsis. A patient with a urinary tract obstruction will be at risk for hydronephrosis. Acute renal failure is not a common complication of acute pyelonephritis unless urosepsis and septic shock develop. Chronic pyelonephritis may occur after recurrent upper UTIs. Cognitive Level: Application Text Reference: p. 1161 Nursing Process: Diagnosis NCLEX: Physiological Integrity

1. When assessing the patient who has a lower urinary tract infection (UTI), the nurse will initially ask about a. flank pain. b. pain with urination. c. poor urine output. d. nausea.

Answer: B Rationale: Pain with urination is a common symptom of a lower UTI. Urine output does not decrease, but frequency may be experienced. Flank pain and nausea are associated with an upper UTI. Cognitive Level: Application Text Reference: p. 1157 Nursing Process: Assessment NCLEX: Physiological Integrity

34. Two days after surgery for an ileal conduit, the patient will not look at the stoma or participate in care. The patient insists that no one but the ostomy nurse specialist care for the stoma. The nurse identifies a nursing diagnosis of a. anxiety related to effects of procedure on lifestyle. b. disturbed body image related to change in body function. c. ineffective health maintenance related to refusal to participate in care. d. self-care deficit, toileting, related to denial of altered body function.

Answer: B Rationale: The patient's unwillingness to look at the stoma or participate in care indicates that disturbed body image is the best diagnosis. There are no data suggesting that the impact on lifestyle is a concern for the patient. The patient may be at risk for ineffective health maintenance if the lack of participation in care continues, but the patient's behavior is normal 2 days after surgery. The patient does not appear to be in denial. Cognitive Level: Application Text Reference: p. 1191 Nursing Process: Diagnosis NCLEX: Psychosocial Integrity

You are teaching your patient diagnosed with myasthenia gravis about treatments. Which of the following statements, if made by the patient indicates the need for further teaching? A) Plasmapheresis is way to reduce symptoms but will need to be done every day B) A thymectomy is a removal of my thymus gland and will show some immediate relieving of my symptoms C) Corticosteroids can be used for short periods of time to help improve my symptoms, but it isn't good for long periods of time D) I need to take my Mestinon four times a day at the same time each day.

Answer: B. A thymectomy may help reduce symptoms, but the effects may not be seen for many months after surgery. Plasmapheresis is the removal of antibodies from blood plasma. It must be done daily for a period of time. Corticosteroids are mostly used for short periods of time unless the patient is experiencing ocular complications. Pyridostigmine bromide (Mestinon) is divided into several doses and should be taken at the same time daily.

8. When admitting a patient with acute glomerulonephritis, the nurse will ask the patient about a. history of high blood pressure. b. frequency of UTIs. c. recent sore throat and fever. d. family history of kidney disease.

Answer: C Rationale: Acute glomerulonephritis frequently occurs after a streptococcal infection such as strep throat. It is not caused by hypertension, UTI, or related to family history. Cognitive Level: Application Text Reference: p. 1165 Nursing Process: Assessment NCLEX: Physiological Integrity

38. Which information noted by the nurse when caring for a patient with a bladder infection is most important to report to the health care provider? a. Dysuria b. Temperature 100.1° F c. Left-sided flank pain d. Hematuria

Answer: C Rationale: Flank pain indicates that the patient may have developed pyelonephritis as a complication of the bladder infection. The other clinical manifestations are consistent with a lower UTI. Cognitive Level: Application Text Reference: p. 1161 Nursing Process: Assessment NCLEX: Physiological Integrity

The patient with myasthenia gravis arrives to the clinic and states that he is experiencing nausea and diarrhea. His blood pressure is 125/85 HR 70 Temp 100.0 R 19 O2 97%. What is the nursing priority? A) Prepare the patient for intubation. He is about to go into a myasthenic crisis. B) Perform teaching on medication side effects C) Assess for signs of infection D) Further assess for other thymectomy complications

Answer: C. Although the GI symptoms is a common side effect of medicaitons, it is important to follow up on the high temperature and assess for sings of infection. An infection can often exacerbate a Myasthenic crisis and should be carefully monitored for. There is no evidence that this person is about to have a myasthenic crisis and intubation should only be done if the patient is experiencing respiratory failure. There is no evidence that this patient has had a thymectomy.

Which of the following is a side effect of Methylprednisolone (Solu-Medrol)? A) Hypovolemia B) Tinnitus C) infection D) Respiratory Depression

Answer: C. Corticosteroid side effects include increased risk for infection, delayed wound healing, infection masking, edema, euphoria, insomnia, hypokalemia, hypocalcemia, hyperglycemia, osteoporosis, and peptic ulcer

Which of the following would be most likely given as a top nursing diagnosis for a patient experiencing a cholinergic crisis? A) Impaired Gas Exchange B) Acute Fatigue C) Ineffective airway clearance D) Altered mental status

Answer: C. During a cholinergic crisis, secretions are increased and the gag reflex is decreased, putting the patient at risk for a blocked airway. Impaired gas exchange, while has to do with respiratory, is not as appropriate as ineffective airway clearance based on the problems of the crisis. Acute fatigue and altered mental status are not priorities.

The nurse is teaching a client about myasthenia gravis. Which statement, if made by the patient indicates the need for further teaching? A) The doctor will take me off of my beta blocker because it could exacerbate my symptoms B) I should report any signs of infection to my PCP C) I can take a ibuprofen to help with pain that may occur with spasms D) I should avoid taking long walks

Answer: C. OTC medication should be avoided as they may worsen MG symptoms. The doctor may stop a beta blocker as they can exacerbate symptoms (unless benefit outweighs the risk). Any signs of infection should be reported as they can exacerbate a myasthenic crisis. Long walks should be avoided due to muscle weakness and fatigue

1. To determine the severity of the symptoms for a patient with benign prostatic hyperplasia (BPH), the nurse will ask the patient about a. the presence of blood in the urine. b. any erectile dysfunction (ED). c. occurrence of a weak urinary stream. d. lower back and hip pain.

Answer: CRationale: The American Urological Association (AUA) Symptom Index for a patient with BPH asks questions about the force and frequency of urination, nocturia, etc. Blood in the urine, ED, and back or hip pain are not typical symptoms with BPH.

6. A 72-year-old patient with benign prostatic hyperplasia and a history of frequent UTIs is admitted to the hospital with chills, fever, and nausea and vomiting. To determine whether the patient has an upper UTI, the nurse will assess for a. suprapubic pain. b. foul-smelling urine. c. bladder distension. d. costovertebral angle (CVA) tenderness.

Answer: D Rationale: CVA tenderness is characteristic of pyelonephritis. The other symptoms are characteristic of lower UTI and are likely to be present if the patient also has an upper UTI. Cognitive Level: Application Text Reference: p. 1161 Nursing Process: Assessment NCLEX: Physiological Integrity

The nurse is caring for a patient with Multiple Sclerosis (MS) and appropriately plans to: A) Teach the patient to avoid all forms of weight bearing exercise B) Avoid the use of an eyepatch as this could cause further damage to vision C) Encourage the patient to consume a low-residue diet D) Teach the patient how to inject medications as all MS medications are administered via SQ or IM injection

Answer: D. It is important for the patient to understand how to inject medication as all MS medication is required to be injected. Weight bearing exercise should be done in moderation and may help with muscle spasticity and prevention of joint contractures. An eyepatch may be beneficial the patient experiencing diplopia. A low-residue diet is low in fiber - patients with MS should consume adequate amounts of fiber to prevent constipation.

Your patient has just been diagnosed with myasthenia gravis. Which of the following orders should be questioned? A) Prednisone PO daily B) Eyepatch to be worn every night C) Pyrodostigmine bromide (Mestinon) 4 times daily PO D) Procaine (Novocain) SQ stat to reduce pain in lower limb

Answer: D. Novocain is contraindicated in patients with MG because of its long lasting effects.

The patient with myasthenia gravis is complaining about dealing with muscle weakness. Which of the following could the nurse do for this patient? A) Administer antispasmodic medication B) Teach the patient to do physical exercise for several hours each day to help strengthen muscles C) Teach the patient it is important to avoid all forms of physical activity whenever possible D) Help the patient form a plan to take medications on time

Answer: D. Taking medications at the same time each day will help reduce the exacerbation of muscle weakness. Antispasmodic medications are not indicated for this patient. Exercising for that much time each day will worsen muscle weakness and fatigue and is not feasible. The patient does not need to avoid all forms of physical activity. They need to time out physical activity with peaks of the medication in order to conserve energy.

A patient with multiple sclerosis states "After I started taking my medication, I feel nauseous and feel fatigued. I also am also running a fever". After looking in the patient's chart you note that she is taking Interferon beta 1b (Betaseron). What is the nurses' best response? A) "We are going to stop your medication immediately. This is a sign of an adverse reaction" B) "It would probably be best to admit you to the hospital. Your MS is relapsing and we will need to begin you on a corticosteroid regimen" C) "This is only a side effect of your medication. It will just eventually go away" D) "Taking your medication at bedtime with a Tylenol my help reduce these symptoms"

Answer: D. Taking the medication at bedtime and managing symptoms with ibuprofen or acetaminophen can help reduce the side-effects of this medication. While option C is true, it is not the most therapeutic response. It offers no suggestion for management. A and B are incorrect because the symptoms are not a sign of adverse reaction or relapsing MS.

Your patient has been diagnosed with MS. You are teaching her about how to reduce muscle spasticity. Which of the following statements, if made by the patient would indicate the need for further teaching? A) Daily exercise, including weight bearing can help relieve spasticity B) My stretching routine can help with the spasms C) Taking Baclofen may help relieve these painful spasms in my legs D) At the end of a day, taking a nice hot bath may relieve the muscle spasms

Answer: D. The patient with MS should never use hot water for a bath due to sensory deficits. All other answers can help with muscle spasms. Warm compresses can be used to relieve muscle spasms.

A pt with HIV is admitted to the hospital with reports of bloody sputum, feeling very tired, night sweats, SOB and has a temp of 99.8F. The nurse recognizes these assessment findings as A. Asthma B. Tuberculosis C. Superinfection resulting from a low CD4 count D. Chronic brochitis

B

When assessing a client diagnosed with left-sided heart failure, the nurse anticipates which finding?A) Abdominal distentionB) Shortness of breathC) Liver enlargementD) Edema of the feet and ankles

B

Which ocular or facial signs/ symptoms should the nurse expect to assess for the client diagnosed with myasthenia gravis? A) Weakness & Fatigue B) Ptosis & diplopia C) Breathlessness & dyspnea D) Weight loss & Dehydration

B

INH treatment is associated with the development of peripheral neuropathies. Which of the following interventions would the nurse teach the client to help prevent this complication? A. Adhere to a low cholesterol diet B. Supplement the diet with pyridoxine (vitamin B6) C. Get extra rest D. Avoid excessive sun exposure

B INH competes with the available vitamin B6 in the body and leaves the client at risk for development of neuropathies related to vitamin deficiency. Supplemental vitamin B6 is routinely prescribed.

When administering furosemide (Lasix) to a client who does not like bananas or orange juice, the nurse recommends that the client try which intervention to maintain potassium levels? A. Increase red meat in the diet. B. Consume melons and baked potatoes. C. Add several portions of dairy products each day. D. Try replacing your usual breakfast with oatmeal or Cream of Wheat.

B Melons and baked potatoes contain potassium. Red meat is high in saturated fat and is to be consumed sparingly. Dairy products are high in calcium. Cereals are fortified with iron; oatmeal contains fiber but not potassium

The nurse has just been assigned the client with pneumonia caused by aspiration after alcohol intoxication recently admitted. The client is agitated and febrile. Which physician order is the nurse's priority? A. Administer the banana bag IV route. B. Draw aerobic and anaerobic blood cultures. C. Give lorazepam (Ativan) as needed for agitation. D. Administer levofloxacin (Levaquin) 500 mg IV.

B Obtaining aerobic and anaerobic cultures is the first action the nurse should perform and is standard procedure in a febrile client for whom antibiotics have been requested. Levofloxacin, an antibiotic, is important to administer, but blood cultures should be drawn before antibiotics are started. Unless this client is a danger to self or staff, giving lorazepam (Ativan) for agitation is not the first action; the question indicates that the client is agitated but does not indicate whether other attempts to control agitation have been tried, such as decreasing stimulation.

This TB test is used in acute care settings to test a symptomatic patient, with results being available within 24 hours A. Nucleic Acid Amplification Test (NAAT) B. Quantiferon-TB Gold (QFT-G) C. Mantoux test D. CXR

B Quantiferon-TB Gold (QFT-G)

A nurse evaluates the blood theophylline level of a client receiving aminophylline (theophylline) by intravenous infusion. The nurse would determine that a therapeutic blood level exists if which of the following were noted in the laboratory report? A. 5 mcg/mL B. 15 mcg/mL C. 25 mcg/mL D. 30 mcg/mL

B The therapeutic theophylline blood level range from 10-20 mcg/mL.

Which pt is at higher risk for developing pneumonia? A. any hospitalized pt between 19 - 64 y.o. B. 36 y.o. trauma pt on mechanical ventilator C. disabled 51 y.o. with abdominal pain, d/c home D. Any pt who has not received the pneumonia vaccine

B 36 y.o. trauma pt on mechanical ventilator

Which of these clients should the charge nurse assign to the LPN/LVN working on the medical-surgical unit? A) Client with group A beta-hemolytic streptococcal pharyngitis who has stridor B) Client with pulmonary tuberculosis who is receiving multiple medications C) Client with sinusitis who has just arrived after having endoscopic sinus surgery D) Client with tonsillitis who has a thick-sounding voice and difficulty swallowing

B) Client with pulmonary tuberculosis who is receiving multiple medications

A client with pneumonia caused by aspiration after alcohol intoxication has just been admitted. The client is febrile and agitated. Which health care provider order should the nurse implement first? A) Administer levofloxacin (Levaquin) 500 mg IV. B) Draw aerobic and anaerobic blood cultures. C) Give lorazepam (Ativan) as needed for agitation. D) Refer to social worker for alcohol counseling.

B) Draw aerobic and anaerobic blood cultures.

pneumonia may present differently in the older adult than in the younger adult? A) Crackles on auscultation B) Fever C) Headache D) Wheezing

B) Fever

a client with PUD reports being nauseated most of the day and now feeling light-headed and dizzy. based upon these findings, which nursing actions would be MOST appropriate for the nurse to take? select all that apply. a. administering an antacid hourly until nausea subsides b. monitoring the client's vital signs c. notifying the physician of the client's symptoms d. initiating oxygen therapy e. reassessing the client in an hour

B, C

Which one of the following are incorrect about TB? Select All That Apply A. The bacterium that causes TB is aerobic, rod shaped and secretes niacin B. Primarily affects the pulmonary system, especially the lower lobes. C. The goal of treatment is to cure the TB D. It is spread via the airborne route E. It is an acid-fast strain

B, C It does primarily affect the pulmo system, but it especially affect the UPPER lobes, where O2 content is highest The goal of treatment is to prevent transmission, control symptoms, and prevent progression of the disease

A patient with COPD needs instruction in measures to prevent pneumonia. What information does the nurse include? (Select all that apply.) A. Avoid going outside B. Clean all respiratory equipment you have at home C. Avoid indoor pollutants such as dust and aerosols D. Get plenty of rest and sleep daily E. Limit alcoholic beverages to 4 to 5 per day

B, C, D

The nurse is caring for a TB pt. She knows that which statements about TB pt care are true Select All That Apply A. HCPs must wear a mask covering face and mouth B. Negative airflow room is required C. HCP must wear an N95 or HEPA mask D. Gloves and Gown are included in PPEs for this pt E. Strict contact precautions must be maintained at all times

B, C, D

Which of the following are correct for the nurse to teach her patient in regard to preventing a UTI? Select all that apply: A. Limit fluid intake B. Void frequently C. Wipe front to back D. Take baths instead of showers E. Urinate after sex F. Void completely

B, C, E, F

the nurse should assess the client who is being admitted to the hospital with upper GI bleeding for which of the following? select all that apply. a. dry, flushed skin b. decreased urine output c. tachycardia d. widening pulse pressure e. rapid respirations f. thirst

B, C, E, F

The pt with advanced cirrhosis asks why his abdomen is so swollen. The nurse's best response is based on the knowledge that a. a lack of clotting factors promotes the collection of blood in the abdominal cavity b. portal hypertension and hypoalbuminemia cause fluid shift into the peritoneal space. c. decreased peristalsis in the GI tract contributes to gas formation and distention of the bowel d. bile salts in the blood irritate the peritoneal membranes, causing edema and pocketing of fluid.

B- Ascites is accumulation of serious fluid in peritoneal cavity. With portal hypertension, protein shifts from the blood into the lymph. When the lymph system is unable to carry excess, it leaks thru the liver into the peritoneal cavity. osmotic pressure of the proteins pulls additional fluid into cavity. Second mechanism of ascites if hypoalbuminemia from the liver unable to synthesize albumin, resulting in decreased colloidal oncotic pressure.

A client with acute renal failure moves into the diuretic phase after one week of therapy. For which signs during this phase should the nurse assess the client? Select all that apply. 1. Dehydration 2. Hypovolemia 3. Hyperkalemia 4. Metabolic acidosis 5. Skin rash A. 1, 2, 3, 4, 5 B. 1, 2 C. 1, 4, 5 D. 1, 2, 3

B. 1, 2

A client with gastroesophageal reflux disease (GERD) should make diet and lifestyle changes. Which instructions should the nurse include in the client's discharge teaching? Select all that apply. 1. Encourage to quit smoking 2. Elevate the foot of the bed 3. Avoid caffeine-containing products 4. Eat three large, evenly spaced meals daily 5. Avoid lying down for 2 to 3 hours after eating A. 1, 2, 3, 4, 5 B. 1, 3, 5 C. 2, 3, 5 D. 1, 4, 5

B. 1, 3, 5

A patient presents with a suspected exacerbation of ulcerative colitis which of the following symptoms would you expect them to say? Select all that apply 1. "I can't catch my breath." 2. "I've noticed blood in my stool." 3. "All of this started after by 75th birthday party." 4. "I have the most pain right before I go to the bathroom" 5. "I feel like I'm going to the bathroom at least 10 times a day, almost every hour." A. 1, 2, 3 B. 2, 4, 5 C. 2, 3, 4 D. 3, 4, 5

B. 2, 4, 5

A pulmonary nurse cares for patients who have chronic obstructive pulmonary disease (COPD). Which patient should the nurse assess first? A. A 46-year-old with a 30-pack-year history of smoking B. A 52-year-old in a tripod position using accessory muscles to breathe C. A 68-year-old who has dependent edema and clubbed fingers D. A 74-year-old with a chronic cough and thick, tenacious secretions

B. A 52-year-old in a tripod position using accessory muscles to breathe

Which assessment data indicate to the nurse the clients gastric ulcer has perforated? A. Complaints of sudden, sharp, substernal pain B. Rigid, boardlike abdomen with rebound tenderness C. Frequent, clay-colored, liquid stool D. Complaints of vague abdominal pain in the right upper quadrant

B. A rigid, boardlike abdomen with rebound tenderness is the classic sign/symptom of peritonitis, which is a complication of a perforated gastric ulcer

A client is admitted to the hospital from the emergency department with a diagnosis of urolithiasis. The nurse reviews the client's clinical record and performs an admission assessment. What is the priority nursing action? A. Strain the client's urine. B. Administer the prescribed morphine. C. Place in the high-Fowler position. D. Collect a urine specimen for culture and sensitivity.

B. Administer the prescribed morphine.

While caring for an obese client who underwent a cholecystectomy, the nurse notices a separation in the surgical incision. Which complication does the nurse identify? A. Adhesions B. Dehiscence C. Evisceration D. Contractions

B. Dehiscence

A patient in the clinic reports a recent episode of dysphasia and left-sided weakness at home that resolved after 2 hours. The nurse will anticipate teaching the patient about a. alteplase (tPA). b. aspirin (Ecotrin). c. warfarin (Coumadin). d. nimodipine (Nimotop).

B. Following a transient ischemic attack (TIA), patients typically are started on medications such as aspirin to inhibit platelet function and decrease stroke risk. tPA is used for acute ischemic stroke. Coumadin is usually used for patients with atrial fibrillation. Nimodipine is used to prevent cerebral vasospasm after a subarachnoid hemorrhage.

You are teaching a patient about post-EGD considerations. Which of the following indicates a need for further teaching? Select all that apply A. A sore throat and belching is normal after this procedure B. I can eat my favorite foods such as carbonated beverages, tomatoes, and dairy as often as I want C. I will need someone to drive me home after the procedure D. I should contact my doctor immediately if I have a fever E. I can continue taking Advil up until the day of my procedure

B. I can eat my favorite foods such as carbonated beverages, tomatoes, and dairy as often as I want E. I can continue taking Advil up until the day of my procedure

With peripheral arterial insufficiency, leg pain during rest can be reduced by: a. Elevating the limb above heart level b. Lowering the limb so it is dependent c. Massaging the limb after application of cold compresses d. Placing the limb in a plane horizontal to the body

B. Lowering the limb so it is dependent

You suspect Bell's palsy in which patient? A. Unilateral facial droop with contralateral extremity weakness B. Sudden onset one-sided facial weakness with ear pain and vesicles C. Sharp, knife-like facial pain when eating hot or cold foods D. Inability to shrug the shoulders against resistance

B. Sudden onset one-sided facial weakness with ear pain and vesiclesBell's palsy is an acute, peripheral facial paresis of unknown cause without systemic effects. Facial droop is found in stroke. Sharp facial pain occurs with trigeminal neuralgia. An inability to shrug the shoulders describes pathology of cranial nerve XI.

The home health nurse is caring for an 81-year-old who had a stroke 2 months ago. Based on information shown in the accompanying figure from the history, physical assessment, and physical therapy/occupational therapy, which nursing diagnosis is the highest priority for this patient? a. Impaired transfer ability b. Risk for caregiver role strain c. Ineffective health maintenance d. Risk for unstable blood glucose level

B. The spouse's household and patient care responsibilities, in combination with chronic illnesses, indicate a high risk for caregiver role strain. The nurse should further assess the situation and take appropriate actions. The data about the control of the patient's diabetes indicates that ineffective health maintenance and risk for unstable blood glucose are not priority concerns at this time. Because the patient is able to ambulate with a cane, the nursing diagnosis of impaired transfer ability is not supported.

Which specific data should the nurse obtain from the client who is suspected of having peptic ulcer disease? A. History of side effects experienced from all medications B. Use of non steroidal anti inflammatory drugs (NSAIDs) C. Any known allergies to drugs and environmental factors D. Medical histories of at lease 3 generations

B. Use of NSAIDs places the client at risk for peptic ulcer and hemorrhage. NSAIDs suppress the production of prostaglandin in the stomach, which is a protective mechanism to prevent damage from hydrochloric acid.

the nurse finds a client who has been diagnosed with a peptic ulcer surrounded by papers from a briefcase and arguing on the telephone with a coworker. the nurse's response to observing these actions should be based on knowledge that: a. involvement with the job will keep the client from becoming bored b. a relaxed environment will promote ulcer healing c. not keeping up with the job will increase the client's stress level d. setting limits on the client's behavior is an important nursing responsibility

B. a relaxed environment will promote ulcer healing

which of the following instructions should the nurse include in the teaching plan for a client who is experiencing GERD a. limit caffeine intake to two cups of coffee per day b. do not lie down for 2 hours after eating c. follow a low-protein diet d. take medications with milk to decrease irritation

B. do not lie down or 2 hours after eating

which of the following would be an expected outcome for a client with PUD? the client will: a. demonstrate appropriate use of analgesics to control pain b. explain the rationale for eliminating alcohol from the diet c. verbalize the importance of monitoring hemoglobin and hematocrit every 3 months d. eliminate engaging in contact sports

B. explain the rationale for eliminating alcohol from the diet

A diagnosis of a ruptured cerebral aneurysm has been made in a patient with manifestations of a stroke. The nurse anticipates that treatment options that would be evaluated for the patient include a. hyperventilation therapy b. surgical clipping of the aneurysm c. administration of hyperosmotic agents d. administration of thrombolytic therapy

B: Surgical clipping of they aneurysm- Surgical management with clipping of an aneurysm to decrease re bleeding and vasospasm is an option for a stroke cause by rupture of a cerebral aneurysm. Placement of coils into the lumens of the aneurysm by intercentional radiologists is increasing in popularity. Hyperventilation therapy would increase vasodilation and the potential for hemorrhage. Thrombolytic therapy would be absolutely contraindicated, and if a vessel is patent, osmotic diuretics may leak into tissue, pulling fluid out of the vessel and increasing edema.

To promote communication during rehabilitation of the patient with aphasia, an appropriate nursing intervention is to a. use gestures, pictures, and music to stimulate patient responses b. talk about activities of daily living (ADLs) that are familiar to the patient c. structure statements so that patient does not have to respond verbally d. use flashcards with simple words and pictures to promote language recall

B: Talk about ADLs that are familiar to the patient- during rehabilitation, the patient with aphasia needs frequent, meaningful verbal stimulation that has relevance for him. Conversation by the nurse and family should address ADLs that are familiar to the patient. Gestures, pictures, and simple statements are more appropriate in the acute phase, when patients may be overwhelmed with verbal stimuli. Flashcards are often perceived by the patient as childish and meaningless.

Which of the following clinical manifestations of type 2 diabetes occurs if glucose levels are very high? Oliguria Increased energy Hyperactivity Blurred vision

Blurred vision

A client comes to the emergency department with a productive cough. Which symptom does the nurse look for that will require immediate attention?A. Blood in the sputumB. Mucoid sputumC. Pink frothy sputumD. Yellow sputum

C

A patient who has a history of pulmonary valve stenosis tells the healthcare provider, "I don't have a lot of energy anymore, and both of my feet get swollen in the late afternoon." Which of these problems does the healthcare provider conclude is the likely cause of these clinical findings? A Acute pericarditis B Peripheral artery disease C Right ventricular failure D Deep vein thrombosis (DVT)

C

Which of the following symptoms might a client with right-sided heart failure exhibit?A Adequate urine outputB PolyuriaC OliguriaD Polydipsia

C

The medical-surgical unit has one negative-airflow room. Which of these four clients who have just arrived on the unit should the charge nurse admit to this room? A. Client with bacterial pneumonia and a cough productive of green sputum B. Client with neutropenia and pneumonia caused by Candida albicans C. Client with possible pulmonary tuberculosis who currently has hemoptysis D. Client with right empyema who has a chest tube and a fever of 103.2° F

C A client with possible tuberculosis should be admitted to the negative-airflow room to prevent airborne transmission of tuberculosis. A client with bacterial pneumonia does not require a negative-airflow room but should be placed in Droplet Precautions. A client with neutropenia should be in a room with positive airflow. The client with a right empyema who also has a chest tube and a fever should be placed in Contact Precautions but does not require a negative-airflow room.

Essential hypertension would be diagnosed in a 40-year-old male whose blood pressure readings were consistently at or above which of the following? A. 120/ 90 mm Hg. B. 130/ 85 mm Hg. C. 140/ 90 mm Hg. D. 160/ 80 mm Hg.

C American Heart Association standards define hypertension as a consistent systolic blood pressure level greater than 140 mm Hg and a consistent diastolic blood pressure level greater than 90 mm Hg.

Which teaching point does the nurse include for a client with peripheral arterial disease (PAD)? A. "Elevate your legs above heart level to prevent swelling." B. "Inspect your legs daily for brownish discoloration around the ankles." C. "Walk to the point of leg pain, then rest, resuming when pain stops." D. "Apply a heating pad to the legs if they feel cold."

C Exercise may improve arterial blood flow by building collateral circulation; instruct the client to walk until the point of claudication, stop and rest, and then walk a little farther.

Which of the following does the nurse recognize as a contributing factor to high BP? A. decreased CO B. pulse rate of 100 C. increased afterload D. decreased stroke volume

C Increased afterload=increased PVR and BP = CO x PVR so if PVR increases then BP increases

The client undergoing femoral popliteal bypass states that he is fearful he will lose the limb in the near future. Which response by the nurse is most therapeutic? A. "Are you afraid you will not be able to work?" B. "If you control your diabetes, you can avoid amputation." C. "Your concerns are valid; we can review some steps to limit disease progression." D. "What about the situation concerns you most?"

C It is important to validate the client's concern and offer needed information. Asking the client if he is afraid may identify fear but does not allow the client to discuss his specific concern.

A client with peripheral arterial disease (PAD) has undergone percutaneous transluminal angioplasty (PTA) of the lower extremity. What is essential for the nurse to assess after the procedure? A. Ankle-brachial index B. Dye allergy C. Pedal pulses D. Gag reflex

C Priority nursing care focuses on assessment for bleeding at the arterial puncture site and monitoring for distal pulses. Pulse checks must be assessed postprocedure to detect improvement (stronger pulses) or complications (diminished or absent pulses).

A pt who has recently traveled to Mexico comes to the ED with fatigue, night sweats, lethargy, and a low-grade fever. What is the nurse's first action? A. Contact the physician for tuberculosis (TB) medications. B. Give the pt a TB skin test. C. Place a respiratory mask on the pt. D. Test all family members for TB.

C The concern is that this client has TB. A respiratory mask should be placed on the client immediately.

The pneumonia pt asks the nurse why she needs to draw labs to check his electrolytes. The nurses correct response is A. To monitor for possible acidosis B. To check for elevated WBCs C. To evaluate the sodium level for possible hypernatremia D. To check for possible septicemia

C Because of dehydration from fever, may be hypernatemic/ dehydrated Electrolytes don't show acidosis, that is ABGS It is important to check the WBC count but its not an electrolyte A blood culture would need to be done for septicemia

A patient has been admitted to the cardiac unit with a diagnosis of right ventricular failure. Which of the following assessment findings would the healthcare provider expect to observe? is most likely to be observed by the healthcare provider? A) Fatigue and hemoptysis B) Bradycardia and circumoral cyanosis C) Peripheral edema and jugular vein distension D) Dyspnea and pulmonary crackles

C) Peripheral edema and jugular vein distension

A client who has recently traveled to Vietnam comes to the emergency department with fatigue, lethargy, night sweats, and a low-grade fever. What is the nurse's first action? A) Contact the health care provider for tuberculosis (TB) medications. B) Perform a TB skin test. C) Place a respiratory mask on the client. D) Test all family members for TB.

C) Place a respiratory mask on the client.

A 63-year-old patient who began experiencing right arm and leg weakness is admitted to the emergency department. In which order will the nurse implement these actions included in the stroke protocol? a. Obtain computed tomography (CT) scan without contrast. b. Infuse tissue plasminogen activator (tPA). c. Administer oxygen to keep O2 saturation >95%. d. Use National Institute of Health Stroke Scale to assess patient.

C, D, A, B The initial actions should be those that help with airway, breathing, and circulation. Baseline neurologic assessments should be done next. A CT scan will be needed to rule out hemorrhagic stroke before tPA can be administered.

what should the nurse teach a client about how to avoid the dumping syndrome? select all that apply. a. consume three regularly spaced meals per day b. eat a diet with high-carb foods with each meal c. reduce fluids with meals, but take them between meals d. obtain adequate amounts of protein and fat in each meal e. eat in a relaxing environment

C, D, E

when obtaining a nursing hx from a client with a suspected gastric ulcer, which signs and symptoms should the nurse assess? select all that apply. a. epigastric pain at night b. relief of epigastric pain after eating c. vomiting d. weight loss e. melena

C, D, E

A nurse is assessing a patient for essential hypertension. She will expect him to report which symptom? A. Chest tightness B. Shortness of Breath C. No symptoms to report D. Anxious

C. Primary (essential) HTN is the silent killer and s/s are not obvious

After attempting lifestyle changes with no improvement in the HTN, the nurse should expect the physician to prescribe which medication first? A. Calcium Channel Blocker B. ARB C. Thiazide diuretic D. Renin inhibitor

C. Thiazide diuretic is the first med to give, sometimes will be combined with a beta blocker. This combo is done so a lower dose of each med can be given.

Which information would the nurse include regarding appliance care and maintenance, when teaching a client with a new colostomy? Select all that apply. 1. Change the ostomy pouch on a routine basis. 2. Replace the ostomy wafer weekly or sooner as needed. 3. Remove the ostomy pouch when showering. 4. Empty the ostomy pouch when three-quarters full of stool or gas. 5. Empty the ostomy pouch before exercise and at bedtime. A. 1, 3, 5 B. 2, 3, 5 C. 1, 2, 5 D. 3, 4, 5

C. 1, 2, 5 (Waiting until pouch is more than half full increases the likelihood of the leakage)

Which foods should a patient with calcium stones avoid in their diet? 1. Yogurt 2. Chicken 3. Fruits 4. Spinach 5. Cheese A. 1, 2, 5 B. 1, 3, 5 C. 1, 4, 5 D. 1, 2, 4

C. 1, 4, 5

A 68-year-old patient is being admitted with a possible stroke. Which information from the assessment indicates that the nurse should consult with the health care provider before giving the prescribed aspirin? a. The patient has dysphasia. b. The patient has atrial fibrillation. c. The patient reports that symptoms began with a severe headache. d. The patient has a history of brief episodes of right-sided hemiplegia.

C. A sudden onset headache is typical of a subarachnoid hemorrhage, and aspirin is contraindicated. Atrial fibrillation, dysphasia, and transient ischemic attack (TIA) are not contraindications to aspirin use, so the nurse can administer the aspirin.

The nurse is caring for a patient who has been experiencing stroke symptoms for 60 minutes. Which action can the nurse delegate to a licensed practical/vocational nurse (LPN/LVN)? a. Assess the patient's gag and cough reflexes. b. Determine when the stroke symptoms began. c. Administer the prescribed short-acting insulin. d. Infuse the prescribed IV metoprolol (Lopressor).

C. Administration of subcutaneous medications is included in LPN/LVN education and scope of practice. The other actions require more education and scope of practice and should be done by the registered nurse (RN).

A 72-year-old patient who has a history of a transient ischemic attack (TIA) has an order for aspirin 160 mg daily. When the nurse is administering medications, the patient says, "I don't need the aspirin today. I don't have a fever." Which action should the nurse take? a. Document that the aspirin was refused by the patient. b. Tell the patient that the aspirin is used to prevent a fever. c. Explain that the aspirin is ordered to decrease stroke risk. d. Call the health care provider to clarify the medication order.

C. Aspirin is ordered to prevent stroke in patients who have experienced TIAs. Documentation of the patient's refusal to take the medication is an inadequate response by the nurse. There is no need to clarify the order with the health care provider. The aspirin is not ordered to prevent aches and pains.

A nurse assesses a client with a mechanical bowel obstruction who reports intermittent abdominal pain. An hour later the client reports constant abdominal pain. Which action should the nurse take next? A. Administer intravenous opioid medications. B. Insert a nasogastric tube for decompression. C. Assess the client's bowel sounds.

C. Assess the client's bowel sounds. Assess first-assess for changes from patient's baseline first.

What is essential teaching in treating a patient with Bell's palsy? A. Perform eye exercises to maintain strength. B. Obtain a herpes simplex virus (HSV) immunization. C. Do not abruptly stop the corticosteroids. D. Vigorously massage the area to promote circulation.

C. Do not abruptly stop the corticosteroids.Corticosteroids are usually started immediately. After they are no longer necessary, they should be tapered. Other treatment includes moist heat, gentle massage, and antiviral medications, such as acyclovir (Zovirax). Eye exercises are not indicated. HSV is identified in 70% of infections, but immunization is not beneficial at this point. Antiviral drugs may be used. Vigorous massage can break down tissues, but gentle upward massage has psychologic benefits.

When caring for a patient with a new right-sided homonymous hemianopsia resulting from a stroke, which intervention should the nurse include in the plan of care? a. Apply an eye patch to the right eye. b. Approach the patient from the right side. c. Place objects needed on the patient's left side. d. Teach the patient that the left visual deficit will resolve.

C. During the acute period, the nurse should place objects on the patient's unaffected side. Because there is a visual defect in the right half of each eye, an eye patch is not appropriate. The patient should be approached from the left side. The visual deficit may not resolve, although the patient can learn to compensate for the defect.

A student nurse demonstrates effective learning when they state that the most common cause of pyelonephritis is: A. P. Hylori B. C. difficile C. E. coli D. Treponema Palladium

C. E. coli

Tommy is recovering from an ileostomy procedure in which a portion of the ileum is brought to the abdomen, and all portions of the large intestine are removed. Now that he is recovering, he is wondering which sports he can participate in. Which sport below can he NOT participate in? A. Swimming B. Cross country running C. Football D. Tennis

C. Football (contact sports)

Clinical manifestations of acute glomerulonephritis include which of the following? A. Chills and flank pain B. Oliguria and generalized edema C. Hematuria and proteinuria D. Dysuria and hypotension

C. Hematuria and proteinuria Hematuria and proteinuria indicate acute glomerulonephritis. These finding result from increased permeability of the glomerular membrane due to the antigen-antibody reaction. Generalized edema is seen most often in nephrosis.

Which stroke risk factor for a 48-year-old male patient in the clinic is most important for the nurse to address? a. The patient is 25 pounds above the ideal weight. b. The patient drinks a glass of red wine with dinner daily. c. The patient's usual blood pressure (BP) is 170/94 mm Hg. d. The patient works at a desk and relaxes by watching television.

C. Hypertension is the single most important modifiable risk factor. People who drink more than 1 (for women) or 2 (for men) alcoholic beverages a day may increase risk for hypertension. Physical inactivity and obesity contribute to stroke risk but not as much as hypertension.

A patient comes into the ED with a chief complaint of "not being able to pee lately." What is a primary nursing diagnosis? A. Acute Pain B. Risk for Fluid Deficit C. Impaired Urinary Elimination D. Incomplete Voiding

C. Impaired Urinary Elimination

An 18 y.o. student is admitted with dark urine, fever, and flank pain and is diagnosed with acute glomerulonephritis. Which would most likely be in this student's health history? A. Renal calculi B. Renal trauma C. Recent sore throat D. Family history of acute glomerulonephritis

C. Recent sore throat The most common form of acute glomerulonephritis is caused by group A beta-hemolytic streptococcal infection elsewhere in the body.

Which intervention would be most beneficial in preventing a catheter-associated urinary tract infection in a postoperative client? A. Pouring warm water over the perineum B. Ensuring the patency of the catheter C. Removing the catheter within 24 hours D. Cleaning the catheter insertion site

C. Removing the catheter within 24 hours

A client with ulcerative colitis has experienced frequent severe exacerbations over the past several years. The client is admitted to the hospital with intense pain, severe diarrhea, and cachexia. Which therapeutic course should the nurse expect the primary healthcare provider to explore with this client? A. Intensive psychotherapy B. Continued medical therapy C. Surgical therapy (colectomy) D. Diet therapy (low-residue, high-protein diet)

C. Surgical therapy (colectomy) If medical management fails, surgical therapy is the next logical choice because it removes the affected intestine. Psychotherapy might improve the client's ability to cope with the disease, but it will not solve the physical problems. Continued medical therapy and diet therapy are classic interventions that probably have been tried during prior exacerbations and have failed.

What nursing intervention may help to prevent the complication of pneumonia in a surgical patient? A. Monitoring chest x-rays and WBC counts for early signs of infection B. Monitoring lung sounds every shift and encouraging fluids C. Teaching coughing, deep-breathing exercises, and use of incentive spirometry D. Encouraging hand hygiene among all caregivers, patients, and visitors

C. Teaching coughing, deep-breathing exercises, and use of incentive spirometry

A 70-year-old female patient with left-sided hemiparesis arrives by ambulance to the emergency department. Which action should the nurse take first? a. Monitor the blood pressure. b. Send the patient for a computed tomography (CT) scan. c. Check the respiratory rate and effort. d. Assess the Glasgow Coma Scale score.

C. The initial nursing action should be to assess the airway and take any needed actions to ensure a patent airway. The other activities should take place quickly after the ABCs (airway, breathing, and circulation) are completed.

A 47-year-old patient will attempt oral feedings for the first time since having a stroke. The nurse should assess the gag reflex and then a. order a varied pureed diet. b. assess the patient's appetite. c. assist the patient into a chair. d. offer the patient a sip of juice.

C. The patient should be as upright as possible before attempting feeding to make swallowing easier and decrease aspiration risk. To assess swallowing ability, the nurse should initially offer water or ice to the patient. Pureed diets are not recommended because the texture is too smooth. The patient may have a poor appetite, but the oral feeding should be attempted regardless.

the nurse is preparing to teach a client with a peptic ulcer about the diet that should be followed after discharge. the nurse should explain that the client should eat which of the following? a. bland foods. b. high-protein foods c. any foods that are tolerated d. a glass of milk with each meal

C. any foods that are tolerated

a client is to take one daily dose of ranitidine (Zantac) at home to treat a peptic ulcer. the client understands proper drug administration of ranitidine when the client will take the drug at which of the following times. a. before meals b. with meals c. at bedtime d. when pain occurs

C. at bedtime

a client who has been diagnosed with a GERD has heartburn. to decrease the heartburn, the nurse should instruct the client to eliminate which of the following items from the diet? a. lean beef b. air-popped popcorn c. hot chocolate d. raw vegetables

C. hot chocolate

a client with suspected gastric cancer undergoes an endoscopy of the stomach. which of the following assessments made after the procedure would indicate the development of a potential complication? a. the client has a sore throat b. the client displays signs of sedation c. the client experiences a sudden increase in temperature d. the client demonstrates lack of appetite

C. the client experiences a sudden increase in temperature

Which intervention should the nurse delegate to the LPN when caring for a patient following an acute stroke? a. assess the patient's neurologic status b. assess the patient's gag reflex before beginning feeding c. administer ordered antihypertensives and platelet inhibitors d. teach the patient's caregivers strategies to minimize unilateral neglect

C: Administer ordered antihypertensives and platelet inhibitors- medication administration is within the scope of practice for an LPN. Assessment and teaching are within the scope of practice for the RN.

Which of the following is the best treatment for acute ischemic stroke? a. heparin b. LMWH c. Alteplase d. Eptifibatie e. Warfarin Which of the following is the best treatment for acute ischemic stroke? a. heparin b. LMWH c. Alteplase d. Eptifibatie e. Warfarin

C: Alteplase

A thrombus that develops in a cerebral artery does not always cause a loss of neurologic function because a. the body can dissolve the atherosclerotic plaques as they form b. some tissues of the brain do not require constant blood supply to prevent damage c. circulation through the circle of Willis may provide blood supply to the affected area of the brain d. neurologic deficits occur only when major arteries are occluded by thrombus formation around an atherosclerotic plaque

C: Circulation through the circle of Willis may provide blood supply to the affected area of the brain. The communication between cerebral arteries in the circle of Willing provides a collateral circulation, which may maintain circulation to an area of the brain if its original blood supply is obstructed. ALl areas of the brain require constant blood supply, and atherosclerotic plaques are not readily reversed. Neurologic deficits can result from ischemia cause by many factors.

The incidence of ischemic stroke in patients with TIAs and other risk factors is reduced with administration of a. furosemide (Lasix) b. lovastatin (Mevacor) c. daily low dose aspirin d. nimodipine (Nimotop)

C: Daily low dose aspirin- the administration of antiplatelet agents, such as aspirin, dipyridamole (Persantine), and ticlopdipine (Ticlid), reduces the incidence of stroke in those at risk. Anticoagulants are also used for prevention of embolic strokes but increase the risk for hemorrhage. Diuretics are not indicated for stroke prevention other than for their role in controlling BP, and antilipemic agents have bot been found to have a significant effect on stroke prevention. The calcium channel blocker nimodipine is used in patients with subarachnoid hemorrhage to decrease the effects of vasospasm and minimize tissue damage. P.S. I freaking love you and good luck on the final!!

A patient with a stroke has a right sided hemiplegia. The nurse prepares family members to help control behavior changes seen with this type of stroke by teaching them to a. ignore undesirable behaviors manifested by the patient b. provide directions to the patient verbally in small steps c. distract the patient from inappropriate emotional responses d. supervise all activities before allowing the patient to pursue them independently

C: Distract the patient from inappropriate emotional responses- patients with left-sided brain damage from stroke often experience emotional lability, inappropriate emotional responses, mood swings, and uncontrolled tears or laughter disproportionate or out of context with the situation. The behavior is upsetting and embarrassing to both the patient and the family, and the patient should be distracted to minimize its presence. Patients with right-brain damage often have impulsive, rapid behavior that supervision and direction.

In promoting health maintenance for prevention of strokes, the nurse understands that the highest risk for the most common type of stroke is present in a. African Americans b. women who smoke c.individuals with hypertension and diabetes d. those who are obese with high dietary fat intake

C: Individuals with hypertension and diabetes- The highest risk factors for thrombotic stroke are hypertension and diabetes. African Americans have a higher risk for stroke than do white persons but probably because they have a greater incidence of hypertension. Factors such as obesity, diet high in saturated fats and cholesterol, cigarette smoking, and excessive alcohol use are also risk factors but carry less risk than hypertension.

A patient with right hemisphere stroke has a nursing diagnosis of unilateral neglect related to sensory perceptual deficits. During the patient's rehabilitation, it is important for the nurse to a. avoid positioning the patient on the affected side b. place all objects for care on the patient's unaffected side c. teach the patient to care consciously for the affected side d. protect the affected side from injury with pillows and supports

C: Teach the patient to care consciously for the affected side- unilateral neglect, or neglect syndrome, occurs when the patient with a stroke is unaware of the affected side of the body, which puts the patient at risk for injury. During the acute phase, the affected side is cared for by the nurse with positioning and support, during rehabilitation the patient is taught to care consciously for and attend to the affected side of the body to protect it from injury. Patients may be positioned on the affected side for up to 30 minutes.

The neurologic functions that are affected by a stroke are primarily related to a. the amount of tissue area involved b. the rapidity of onset of symptoms c. the brain area perfused by the affected artery d. the presence or absence of collateral circulation

C: The brain area perfused by the affected artery- clinical manifestation of altered neurologic function differ, depending primarily on the specific cerebral artery involved and the area of the brain that is perfused by the artery. The degree of impairment depends on rapidity of onset, the size of the lesion, and the presence of collateral circulation.

An appropriate food for a patient with a stroke who has mild dysphagia is a. fruit juices b. pureed meat c. scrambled eggs d. fortified milkshakes

C: scrambled eggs- soft foods that provide enough texture, flavor, and bulk to stimulate swallowing should be used for the patient with dysphasia. Thin liquids are difficult to swallow, and patients may not be able to control them in the mouth. Pureed foods are often too bland and to smooth, and milk products should be avoided because they tend to increase the viscosity of mucus and increase salivation.

Which finding about a patient with polycythemia vera is most important for the nurse to report to the health care provider? a. Hematocrit 55% c. Calf swelling and pain b. Presence of plethora d. Platelet count 450,000/L

Calf swelling and pain- DVT

Which of the following clinical characteristics is associated with type 2 diabetes (previously referred to as non-insulin dependent diabetes mellitus [NIDDM])? Usually thin at diagnosis Demonstrate islet cell antibodies Can control blood glucose through diet and exercise Ketosis-prone

Can control blood glucose through diet and exercise

A client's blood glucose level is 45 mg/dl. The nurse should be alert for which signs and symptoms? Coma, anxiety, confusion, headache, and cool, moist skin Polyuria, polydipsia, hypotension, and hypernatremia Kussmaul's respirations, dry skin, hypotension, and bradycardia Polyuria, polydipsia, polyphagia, and weight loss

Coma, anxiety, confusion, headache, and cool, moist skin

A nurse educates a group of clients with diabetes mellitus on the prevention of diabetic nephropathy. Which of the following suggestions would be most important? Control blood glucose levels. Take the antidiabetic drugs regularly. Drink plenty of fluids. Eat a high-fiber diet.

Control blood glucose levels.

A 67-year-old man with peripheral artery disease is seen in the primary care clinic. Which symptom reported by the patient would indicate to the nurse that the patient is experiencing intermittent claudication? a Patient complains of chest pain with strenuous activity. b Patient says muscle leg pain occurs with continued exercise. c Patient has numbness and tingling of all his toes and both feet. d Patient states the feet become red if he puts them in a dependent position.

Correct Answer: B Intermittent claudication is an ischemic muscle ache or pain that is precipitated by a consistent level of exercise, resolves within 10 minutes or less with rest, and is reproducible. Angina is the term used to describe chest pain with exertion. Paresthesia is the term used to describe numbness or tingling in the toes or feet. Reactive hyperemia is the term used to describe redness of the foot; if the limb is in a dependent position the term is dependent rubor.

The client is diagnosed with MG. Which intervention should the nurse implement when administering the anticholinesterase pyridostigmine (Mestinon)? 1. Assess for excess salivation and abdominal cramps. 2. Administer the medication before the client has eaten. 3. Break the capsule and sprinkle the medication on the food. 4. Assess the client's potassium level prior to administering medication.

Correct answer 1: Anticholinesterase medications can cause the client to have excessive salivation and abdominal cramping. When this occurs, the client receives the antidote atropine simultaneously in small doses. Mestinon is administered with milk and/or crackers to prevent stomach upset. Mestinon does not affect potassium levels.

The male client with MG is undergoing plasmapheresis at the bedside. Which assessment data would warrant immediate intervention by the nurse? 1. The client complains of being lightheaded and dizzy. 2. The client can smile and clamp his teeth together. 3. The client states that his leg cramps have gone away. 4. The client has a small hematoma at the vascular access site.

Correct answer 1: Hypovolemia is a complication of plasmapheresis, especially during the procedure when up to 15% of the blood volume is in the cell separator. The nurse should immediately assess for shock. All other options are expected.

Which statement by the client supports the diagnosis of myasthenia gravis (MG)? 1. "I have weakness and fatigue in my feet and legs." 2. "My eyelids droop, and I see double everything." 3. "I get chest pain and faint after I walk in the hall." 4. "I gained 3 pounds this week, and I am spitting up pink frothy sputum."

Correct answer 2: These are ocular signs/symptoms of MG. Ptosis is drooping of the eyelid, and diplopia is unilateral or bilateral double vision. Weakness and fatigue of upper body muscle occur with MG. Option 3 is angina. Option 4 is heart failure.

The client diagnosed with MG is admitted to the emergency department with a sudden exacerbation of motor weakness. Which assessment data indicate the client is experiencing a myasthenic crisis? 1. The serum assay of circulating acetylcholine receptor antibodies is increased. 2. The client's symptoms improve when administering on a cholinesterase inhibitor. 3. The client's blood pressure, pulse, and respirations improve after intravenous (IV) fluid. 4. The Tensilon test does not show improvement in the client's muscle strength.

Correct answer 2: This assessment datum indicates a myasthenic crisis that is due to undermedication, missed doses of medication, or developing an infection. Serum assays are useful in diagnosing the disease, not in identifying a crisis. Vital signs do not differentiate the type of crisis. No improvement after Tensilon indicates a cholinergic crisis, not a myasthenic crisis.

The male client diagnosed with MG is prescribed the cholinesterase inhibitor neostigmine (Prostigmin). Which data indicate the medication is not effective? l 1. The client is able to perform activities of daily living (ADLs) independently. l 2. The client states that his vision is clear. l 3. The client cannot speak or look upward at the ceiling. l 4. The client is smiling and laughing with the nurse.

Correct answer 3: Dysphonia and inability to utilize the muscles of the eye and eyelid indicate the medication is not effective. Performing ADLs, having clear vision, and smiling and laughing using the facial muscles indicate the medication is effective

The nurse is discharging a client diagnosed with MG. Which statement by the client indicates an understanding of the discharge instructions? 1. "I can control the MG with medication, but an adenectomy will cure it." 2. "I should take a holiday from my medications every 4 or 5 weeks." 3. "I must take my medications on time every day, or I could have problems." 4. "I should take my steroid medications with food so it won't upset my stomach."

Correct answer 3: The anti cholinesterase medications used to treat MG must be taken on time in order to prevent muscle weakness and respiratory complications. These medications are one of the very few that the nurse should administer at the exact scheduled time. Steroids are not prescribed for MG.

Which response to the Tensilon (edrophonium chloride) injection indicates the client has myasthenia gravis? 1. The client has no apparent change in the assessment data. 2. There is reduced amplitude of electrical stimulation in the muscle. 3. The anti-acetylcholine receptor antibodies are present. 4. The client shows a marked improvement of muscle strength.

Correct answer 4: Clients with myasthenia gravis show a significant improvement of muscle strength that lasts approximately 5 minutes when Tensilon (edrophonium chloride) is injected.

Which referral is appropriate for the client in the late stages of myasthenia gravis? 1. The infection control nurse. 2. The occupational health nurse. 3. A vocational guidance counselor. 4. The speech therapist.

Correct answer 4: Speech therapists address swallowing problems, and clients with myasthenia gravis are dysphagic and at risk for aspiration. The infection control and occupational health nurses do not consult with the client. A vocational counselor helps with the client finding a position suited for the disability, but clients with late-stage myasthenia gravis are usually not able to work.

23. A hospitalized patient with heart failure has a new order for captopril (Capoten) 12.5 mg PO. After administering the first dose and teaching the patient about captopril, which statement by the patient indicates that teaching has been effective? a. "I will need to include more high-potassium foods in my diet." b. "I will expect to feel more short of breath for the next few days." c. "I will be sure to take the medication after eating something." d. "I will call for help when I need to get up to the bathroom."

D

A patient has been admitted to the cardiac unit with a diagnosis of right ventricular failure. Which of the following assessment findings is most likely to be observed by the healthcare provider? A Bradycardia and circumoral cyanosis B Fatigue and hemoptysis C Dyspnea and pulmonary crackles D Peripheral edema and jugular vein distension

D

A patient is diagnosed with heart failure and is prescribed digoxin (Lanoxin) and furosemide (Lasix). Before administering the furosemide to the patient, which laboratory result should the healthcare provider to review?A Blood urea nitrogen (BUN) B Serum sodium C Serum troponin D Serum potassium

D

An African American male client with a history of diabetes and stroke returns from coronary angioplasty with stent placement. Which priority action will the nurse perform at this time?A) Discontinuing intravenous lines when taking oral fluidsB) Securing chest tubes to the beddingC) Treating chest pain with intravenous morphine as neededD) Maintaining leg extension on the affected side

D

The client is being evaluated to rule out myasthenia gravis & being administered the Tensilon test. Which response to the test indicates the client has MG? A) The client has no apparent change in the assessment data B) This is increased amplitude of electrical stimulation in the muscle C) The circulating acetylcholine receptor antibodies are decreased D) The client shows a marked improvement of muscle strength

D

The client w/ MG is admitted to the emergency room with a sudden exacerbation of motor weakness. Which assessment data indicate the client is experiencing cholinergic crisis? A) The serum assay of circulation ACH receptor antibodies is increased B) The client's symptoms improve when administering cholinesterase inhibitor C) Clients BP, pulse, and RR improve after IV fluid D) The tensilon test does not show improvement in the client's muscle strength

D

The wife of a client diagnosed with MG is crying and shares with the nurse she just doesn't know what to do. Which response is the best action by the nurse? A) Discuss the Myasthenia foundation w/ client's wife B) Refer client to a local MG support group C) Ask the pts. Wife if she would like to talk to a counselor D) Sit down & allow the wife to ventilate her feelings to the nurse

D

The nurse notices a visitor walking into the room of a pt on airborne isolation with no protective gear. What does the nurse do? A. Ensures that the pt is wearing a mask B. Tells the visitor that the pt cannot receive visitors at this time C. Gives a particulate air respirator to the visitor D. Gives a mask to the visitor

D Because the visitor is entering the pt's isolation environment, the visitor must wear a mask. The pt typically must wear a mask only when he or she is outside of an isolation environment. Turning the visitor away is inappropriate and unnecessary. It would not be necessary for the visitor to wear an air respirator.

A client's ABG analysis reveals a pH of 7.18, PaCO2 of 72 mm Hg, PaO2 of 77 mm Hg, and HCO3- of 24 mEq/L. What do these values indicate? A. Metabolic acidosis B. Respiratory alkalosis C. Metabolic alkalosis D. Respiratory acidosis

D Respiratory Acidosis

The nurse is reviewing the lab results for an older adult pneumonia pt. The lab value frequently seen in pneumonia pts that may not be seen in this pt is A. RBC 4.0 - 5.0 B. Hgb 12 - 16 C. Hct 36 - 48 D. WBC 12 - 18

D WBC

The nurse is reviewing a pneumonia pts lab results. What does she expect to see A. decreased Hgb B. increased RBCs C. decreased neutrophils D. increased WBCs

D incr WBCs

A pt is at the office for a follow up visit and has been compliant with drug therapy for TB. Which result indicates the TB is no longer infectious A. a negative chest x-ray B. three negative sputum cultures and a negative chest x-ray C. no clinical symptoms present D. three negative sputum cultures

D three negative sputum cultures

Your patient diagnosed with Myasthenia Gravis begins taking Mestinon. During the first week, the dosage is changed frequently. While the dosage is being adjusted, the nurse's priority intervention is to: A) Administer the medication with food or an 8 oz. glass of water B) Evaluate the client's muscle strength hourly after medication C) Take a full set of vital signs every 15 minutes D) Administer the medication exactly on time

D) Peak response occurs 1 hour after administration and lasts up to 8 hours. By giving the medication exactly on time, this will help determine dosage levels. Mestinon can be given with or without food/water. There is nothing in this question that indicates vitals should be taken every 15 minutes. The client's muscle strength is important to assess, but the priority intervention is to give the medication on time.

When obtaining a health history of a patient admitted with a diagnosis of heart failure, which statement made by the patient supports the diagnosis of heart failure? A) "I often feel pain in my lower legs when I take my walk." B) "I sometimes feel pain in the middle of my chest during exercise." C) "I get hot and break out in a sweat during the night." D) "I get out of breath when I go up a flight of stairs

D) "I get out of breath when I go up a flight of stairs

The nurse suspects that a client has developed an acute arterial occlusion of the right lower extremity based on which signs/symptoms? (Select all that apply.) A. Hypertension B. Tachycardia C. Bounding right pedal pulses D. Cold right foot E. Numbness and tingling of right foot F. Mottling of right foot and lower leg

D, E, F Pain, pallor, pulselessness, paresthesia, paralysis, poikilothermia (cool limb), and mottled color are characteristics of acute arterial occlusion.

When planning care for a pt with cirrhosis, the nurse will give highest priority to which nursing diagnosis? a. impaired skin integrity related to edema, ascites, and pruritis b. imbalanced nutrition: less than body requirements related to anorexia c. excess fluid volume related to portal hypertension and hyperaldosteronism d. ineffective breathing pattern related to pressure on diaphragm and reduced lung volume

D-airway and breathing are always highest priority.

The health care provider orders lactulose for a patient with hepatic encephalopathy. The nurse will monitor for effectiveness of this medication for this pt by assessing what? a. relief of constipation b. relief of ab pain c. decreased liver enzymes d. decreased ammonia levels

D-hepatic encephalopathy is associated with elevated ammonia levels. Lactulose traps ammonia in the intestinal tract. It's laxative effect then expels ammonia from the colon, resulting in decreased ammonia levels, correcting hepatic encephalopathy.

a client is taking an antacid for treatment of a peptic ulcer. which of the following statements best indicates that the client understands how to correctly take the antacid? a. "I should take my antacid before I take my other medications" b. "I need to decrease my intake of fluids so that i don't dilute the effects of my antacid" c. "my antacid will be most effective if i take it whenever i experience stomach pains" d. "it is best for me to take my antacid 1 to 3 hours after meals"

D. "it is best for me to take my antacid 1-3 hours after meals"

What is the priority of care to promote client safety directly after esophagogastroduodenoscopy? Select all that apply. 1. Preventing aspiration 2. Reminding the client not to drive 3. Monitoring for signs of perforation 4. Advising the client to use throat lozenges 5. Teaching the client about hoarseness of voice A. 2, 3 B. 3, 5 C. 4, 5 D. 1, 3

D. 1, 3

When teaching about clopidogrel (Plavix), the nurse will tell the patient with cerebral atherosclerosis a. to monitor and record the blood pressure daily. b. that Plavix will dissolve clots in the cerebral arteries. c. that Plavix will reduce cerebral artery plaque formation. d. to call the health care provider if stools are bloody or tarry.

D. Clopidogrel (Plavix) inhibits platelet function and increases the risk for gastrointestinal bleeding, so patients should be advised to notify the health care provider about any signs of bleeding. The medication does not lower blood pressure, decrease plaque formation, or dissolve clots.

Which meal would be best for a patient who has received a cholecystectomy? A. Fried chicken strips, Cajun French fries, and a milkshake B. Cheeseburger, onion rings, Dr. Pepper C. Avocado and feta cheese wrap, salmon cooked sautéed in olive oil, dark chocolate D. Grilled chicken breast, spinach salad with lemon juice dressing, apple

D. Grilled chicken breast, spinach salad with lemon juice dressing, apple

A nurse evaluates the following arterial blood gas and vital sign results for a patient with COPD: Arterial Blood Gas Results: pH = 7.32 PaCO2 = 62 mm Hg PaO2 = 46 mm Hg HCO3- = 28 mEq/L Vital Signs: Heart rate = 110 beats/min Respiratory rate = 12 breaths/min Blood pressure = 145/65 mm Hg Oxygen saturation = 76% Which action should the nurse take first? A. Administer a short-acting beta2 agonist inhaler. B. Document the findings as normal for a patient with COPD. C. Teach the patient diaphragmatic breathing techniques. D. Initiate oxygenation therapy to increase saturation to 92%.

D. Initiate oxygenation therapy to increase saturation to 92%.

Which problems should the nurse include in the plan of care for the client diagnosed with peptic ulcer disease to observe for physiological complications? A. Alteration in bowel elimination patterns B. Knowledge deficit in the causes of ulcers C. Inability to cope with changing family roles D. Potential for alteration in gastric emptying

D. Potential for alteration in gastric emptying is caused by edema or scarring associated with an ulcer, which may cause a feeling of "fullness", vomiting of undigested food or abdominal distention

A patient with left-sided weakness that started 60 minutes earlier is admitted to the emergency department and diagnostic tests are ordered. Which test should be done first? a. Complete blood count (CBC) b. Chest radiograph (Chest x-ray) c. 12-Lead electrocardiogram (ECG) d. Noncontrast computed tomography (CT) scan

D. Rapid screening with a noncontrast CT scan is needed before administration of tissue plasminogen activator (tPA), which must be given within 4.5 hours of the onset of clinical manifestations of the stroke. The sooner the tPA is given, the less brain injury. The other diagnostic tests give information about possible causes of the stroke and do not need to be completed as urgently as the CT scan.

A 73-year-old patient with a stroke experiences facial drooping on the right side and right-sided arm and leg paralysis. When admitting the patient, which clinical manifestation will the nurse expect to find? a. Impulsive behavior b. Right-sided neglect c. Hyperactive left-sided tendon reflexes d. Difficulty comprehending instructions

D. Right-sided paralysis indicates a left-brain stroke, which will lead to difficulty with comprehension and use of language. The left-side reflexes are likely to be intact. Impulsive behavior and neglect are more likely with a right-side stroke.

Which assessment data support to the nurse the clients diagnosis of gastric ulcer? A. Presence of blood in the clients stool for the past month. B. Reports of a burning sensation moving like a wave. C. Sharp pain in the upper abdomen after eating a heavy meal. D. Complaints of epigastric pain 30-60 minutes after ingesting food.

D. The client diagnosed with a gastric ulcer, pain usually occurs 30 to 60 minutes after eating, but not at night. In contrast,no client with a duodenal ulcer has pain during the night often relieved by eating food. Pain occurs 1-3 hours after meals.

a client with a peptic ulcer has been instructed to avoid intense physical activity and stress. which strategy should the client incorporate into the home care plan? a. conduct physical activity in the morning in order to be able to rest in the afternoon b. have the family agree to perform the necessary yard work at home c. give up jogging and substitute a less demanding hobby d. incorporate periods of physical and mental rest in the daily schedule

D. incorporate periods of physical and mental rest in the daily schedule

a client with PUD is taking ranitidine. what is the expected outcome of this drug? a. heal the ulcer b. protect the ulcer surface from acids c. reduce acid concentration d. limit gastric acid secretion

D. limit gastric acid secretion

a client is admitted to the hospital after vomiting bright red blood and is diagnosed with a bleeding duodenal ulcer. the client develops a sudden, sharp pain in the midepigastric region along with a rigid, boardlike abdomen. the nurse should do which of the following first? a. administer pain medication as prescribed b. raise the HOB c. prepare to insert an NG tube d. notify HCP

D. notify HCP

A nursing intervention is indicated for the patient with hemiplegia is a. the use of a footboard to prevent plantar flexion b. immobilization of the affected arm against the chest with a sling c. positioning the patient in bed with each joint lower than the joint proximal to it d. having the patient perform passive ROM of the affected limb with the unaffected limb

D: Having the patient perform passive ROM of the affected limb with the unaffected limb- active ROM should be initiated on the unaffected side as soon as possible, and passive ROM of the affected side should be started on the first day. Having the patient actively exercise the unaffected side provides the patient with active and passive ROM as needed. Use of footboards is controversial because they stimulate plantar flexion. The unaffected arm should be supported, but immobilization may precipitate a painful shoulder-hand syndrome. The patient should be positioned with each joint higher than the joint proximal to it to prevent dependent edema.

The nurse can assist the patient and the family in coping with the long term effects of a stroke by a. informing family members that the patient will need assistance with almost all ADLs b. explaining that the patient's prestroke behavior will return as improvement progresses c. encouraging the patient and family members to seek assistance from family therapy or stroke support groups d. helping the patient and family understand the significance of residual stroke damage to promote problem solving and planning

D: Helping the patient and family understand the significance of residual stroke damage to promote problem solving and planning- the patient and family need accurate and complete information about the effects of the stroke to problem solve and make plans for chronic care of the patient. It is uncommon for patients with major strokes to return completely to pre stroke function, behaviors, and role, and both the patient and family will mourn these losses. The patient's specific needs for care must be identified, and rehabilitation efforts should be continued at home. Family therapy and support groups may be helpful for some patients and families.

The priority intervention in the emergency department for the patient with a stroke is a. intravenous fluid replacement b. administration of osmotic diuretics to reduce cerebral edema c. initiation of hypothermia to decrease the oxygen needs of the brain d. maintenance of respiratory function with a patent airway and oxygen administration

D: Maintenance of respiratory function with a patent airway and oxygen administration- the first priority in acute management of the patient with a stroke is preservation of life. Because the patient with a stroke may be unconscious or have a reduced gag reflex, it is most important to maintain a patent airway for the patient and provide oxygen if respiratory effort is impaired. IV fluid replacement, treatment with osmotic diuretics, and perhaps hypothermia may be used for further treatment.

A patient's wife asks the nurse why her husband did not receive the clot busting medication (tPA) she has been reading about. Her husband is diagnosed with a hemorrhagic stroke. What should the nurse respond? a. He didn't arrive within the time frame for that therapy b. Not every is eligible for this drug. Has he had surgery lately? c. You should discuss the treatment of your husband with your doctor d. The medication you are talking about dissolves clots and could cause more bleeding in your husband's head

D: The medication you are talking about dissolves clots and could cause more bleeding in your husband's head- tPA dissolves clots and increases the risk for bleeding. It is not used with hemorrhagic strokes. If the patient had a thrombotic/embolic stroke the time frame would be important as well as a history of surgery. The nurse should answer the question as accurately as possible and then encourage the individual to talk with the primary care physician if he or she has further questions.

A patient comes to the emergency department immediately after experiencing numbness of the face and an inability to speak, but while the patient awaits examination, the symptoms disappear and the patient request discharge. The nurse stresses that it is important for the patient to be evaluated primarily because a. the patient has probably experienced an asymptomatic lacunar stroke b. the symptoms are likely to return and progress to worsening neurologic deficit in the next 24 hours c. neurologic deficits that are transient occur most often as a result of small hemorrhages that clot off d. the patient has probably experienced a transient ischemic attack (TIA), which is a sign of progressive cerebral vascular disease

D: The patient has probably experienced a transient ischemic attack (TIA), which is a sign of progressive cerebral vascular disease- A TIA is a temporary focal loss of neurologic function caused by ischemia of an area of the brain, usually lasting only about 3 hours. TIAs may be due to microemboli from heart disease or carotid or cerebral thrombi and are a warning of progressive disease. Evaluation is necessary to determine the cause of the neurologic deficit and provide prophylactic treatment if possible.

A client with type 1 diabetes mellitus is receiving short-acting insulin to maintain control of blood glucose levels. In providing glucometer instructions, the nurse would instruct the client to use which site for most accurate findings? Upper arm Forearm Thigh Finger

Finger

A nurse is teaching a client with type 1 diabetes how to treat adverse reactions to insulin. To reverse hypoglycemia, the client ideally should ingest an oral carbohydrate. However, this treatment isn't always possible or safe. Therefore, the nurse should advise the client to keep which alternate treatment on hand? Hydrocortisone 50% dextrose Epinephrine Glucagon

Glucagon

Which of the following is the most rapid acting insulin? Humalog Ultralente NPH Regular

Humalog

A patient with diabetes mellitus is receiving an oral antidiabetic agent. The nurse observes for which of the following symptoms when caring for this patient? Hypoglycemia Polyuria Polydipsia Blurred vision

Hypoglycemia

When the nurse is caring for a patient with type 1 diabetes, what clinical manifestation would be a priority to closely monitor? Polyphagia Hypoglycemia Hyponatremia Ketonuria

Hypoglycemia

Which combination of adverse effects should a nurse monitor for when administering I.V. insulin to a client with diabetic ketoacidosis? Hyperkalemia and hyperglycemia Hypernatremia and hypercalcemia Hypokalemia and hypoglycemia Hypocalcemia and hyperkalemia

Hypokalemia and hypoglycemia

The nurse is educating the diabetic client on setting up a sick plan to manage blood glucose control during times of minor illness such as influenza. Which is the most important teaching item to include? Do not take insulin if not eating. Increase frequency of glucose self-monitoring. Decrease food intake until nausea passes. Take half the usual dose of insulin until symptoms resolve.

Increase frequency of glucose self-monitoring.

A nurse is providing education to a client who is newly diagnosed with diabetes mellitus. Which of the following symptoms would she include when reviewing classic symptoms associated with diabetes? Increased weight gain, increased appetite, and increased thirst Increased weight loss, increased dehydration, and increased fatigue Loss of appetite, increased urination, and dehydration Increased thirst, increased hunger, and increased urination

Increased thirst, increased hunger, and increased urination

NPH is an example of which type of insulin? Intermediate-acting Long-acting Short-acting Rapid-acting

Intermediate-acting

The nurse is describing the action of insulin in the body to a client newly diagnosed with type 1 diabetes. Which of the following would the nurse explain as being the primary action? It aids in the process of gluconeogenesis. It decreases the intestinal absorption of glucose. It enhances transport of glucose across the cell wall. It stimulates the pancreatic beta cells.

It enhances transport of glucose across the cell wall.

A 53-year-old client is brought to the ED, via squad, where you practice nursing. He is demonstrating fast, deep, labored breathing and has a fruity odor to his breath. He has a history of type 1 diabetes. What could be the cause of his current serious condition? Hyperosmolar hyperglycemic nonketotic syndrome Hepatic disorder All options are correct Ketoacidosis

Ketoacidosis

Which of the following are byproducts of fat breakdown, which accumulate in the blood and urine? Ketones Creatinine Cholesterol Hemoglobin

Ketones

The nurse is reviewing the lab results for a pt with cirrhosis and notes that the ammonia level is elevated. Which diet does the nurse anticipate to be prescribed for this pt? Low-protein High-protein Moderate-fat High-carb

Low-protein dietProtein provided by the diet is transported to the liver via the portal vein. The liver breaks down protein, which results in the formation of ammonia.

A client with diabetes is receiving an oral anti diabetic agent that acts to help the tissues use available insulin more efficiently. Which of the following agents would the nurse expect to administer? Glipizide Glyburide Metformin Repaglinide

Metformin

A nurse expects to find which signs and symptoms in a client experiencing hypoglycemia? Nervousness, diaphoresis, and confusion Polyuria, headache, and fatigue Polydipsia, pallor, and irritability Polyphagia and flushed, dry skin

Nervousness, diaphoresis, and confusion

For patients who are newly diagnosed with Obstructive Sleep Apnea, what is a major consideration that the nurse should prioritize in the management of OSA? Increased appetite Depression Patient's ability to adapt to therapy Having a new mattress and pillows

Patient's ability to adapt to therapy

Lispro (Humalog) is an example of which type of insulin? Rapid-acting Intermediate-acting Short-acting Long-acting

Rapid-acting

The nurse is administering an insulin drip to a patient in ketoacidosis. What insulin does the nurse know is the only one that can be used intravenously? Lantus Regular Lispro NPH

Regular

A client with diabetes mellitus has a prescription for 5 units of U-100 regular insulin and 25 units of U-100 isophane insulin suspension (NPH) to be taken before breakfast. At about 4:30 p.m., the client experiences headache, sweating, tremor, pallor, and nervousness. What is the most probable cause of these signs and symptoms? Serum calcium level of 8.9 mg/dl Serum calcium level of 10.2 mg/dl Serum glucose level of 52 mg/dl Serum glucose level of 450 mg/dl

Serum glucose level of 52 mg/dl

After being sick for 3 days, a client with a history of diabetes mellitus is admitted to the hospital with diabetic ketoacidosis (DKA). The nurse should evaluate which diagnostic test results to prevent arrhythmias? Serum calcium level Serum chloride level Serum potassium level Serum sodium level

Serum potassium level

A nursing instructor is discussing asthma and its complications with medical-surgical nursing students. Which of the following would the group identify as complications of asthma? Choose all that apply. -Status asthmaticus -Respiratory failure -Pertussis -Atelectasis -Thoracentesis

Status asthmaticus-Respiratory failure-AtelectasisComplications of asthma may include status asthmaticus, respiratory failure, pneumonia, and atelectasis. Pertussis and pneumothorax are not complications.

A client with status asthmaticus requires endotracheal intubation and mechanical ventilation. Twenty-four hours after intubation, the client is started on the insulin infusion protocol. The nurse must monitor the client's blood glucose levels hourly and watch for which early signs and symptoms associated with hypoglycemia? Sweating, tremors, and tachycardia Dry skin, bradycardia, and somnolence Bradycardia, thirst, and anxiety Polyuria, polydipsia, and polyphagia

Sweating, tremors, and tachycardia

A male client, aged 42, is diagnosed with diabetes mellitus. He visits the gym regularly and is a vegetarian. Which of the following factors is important when assessing the client? History of radiographic contrast studies that used iodine The client's consumption of carbohydrates The client's exercise routine The client's mental and emotional status

The client's consumption of carbohydrates

The nurse is teaching the client about use of the pictured item with a metered-dose inhaler (MDI). The nurse instructs the client as follows: (Select all that apply.) -Take a slow, deep inhalation from the device. -Use normal inhalations with the device. -Activate the MDI once. -The device may increase delivery of the MDI medication. -It is not necessary to hold your breath after using.

The nurse is teaching the client about use of the pictured item with a metered-dose inhaler (MDI). The nurse instructs the client as follows: (Select all that apply.)-Take a slow, deep inhalation from the device.-Use normal inhalations with the device.-Activate the MDI once.-The device may increase delivery of the MDI medication.-It is not necessary to hold your breath after using.

A nurse is preparing to administer two types of insulin to a client with diabetes mellitus. Which of the following demonstrates that the nurse understands the correct procedure for preparing this medication? The short-acting insulin is withdrawn before the intermediate-acting insulin. Different types of insulin are not to be mixed in the same syringe. The intermediate-acting insulin is withdrawn before the short-acting insulin. If administered immediately, there is no requirement for withdrawing one type of insulin before another.

The short-acting insulin is withdrawn before the intermediate-acting insulin.

A client with diabetes mellitus develops sinusitis and otitis media accompanied by a temperature of 100.8° F (38.2° C). What effect do these findings have on his need for insulin? They cause wide fluctuations in the need for insulin. They increase the need for insulin. They decrease the need for insulin. They have no effect.

They increase the need for insulin.

Which intervention is essential when performing dressing changes on a client with a diabetic foot ulcer? Using sterile technique during the dressing change Cleaning the wound with a povidone-iodine solution Applying a heating pad Debriding the wound three times per day

Using sterile technique during the dressing change

The nurse is teaching a patient about self-administration of insulin and mixing of regular and neutral protamine Hagedorn (NPH) insulin. Which of the following is important to include in the teaching plan? When mixing insulin, the NPH insulin is drawn up into the syringe first. If two different types of insulin are ordered, they need to be given in separate injections. When mixing insulin, the regular insulin is drawn up into the syringe first. There is no longer a need to inject air into the bottle of insulin before insulin is withdrawn.

When mixing insulin, the regular insulin is drawn up into the syringe first.

1) The client enters the outpatient clinic and states to the triage nurse, "I think I have the flu. I'm so tired, I have no appetite, and everything hurts." The triage nurse assesses the client and finds a butterfly rash over the bridge of nose and on the cheeks. Which diagnosis does the nurse expect? A) Systemic lupus erythematosus B) Fibromyalgia C) Lyme disease D) Gout

a

A 45-year-old man with asthma is brought to the emergency department by automobile. He is short of breath and appears frightened. During the initial nursing assessment, which clinical manifestation might be present as an early manifestation during an exacerbation of asthma? Anxiety Cyanosis Bradycardia Hypercapnia

a

A patient scheduled for a transurethral resection of the prostate (TURP) for BPH tells the nurse that he has delayed having surgery because he is afraid it will affect his sexual function. When responding to his concern, the nurse explains that a. with this type of surgery, erectile problems are rare, but retrograde ejaculation may occur. b. information about penile implants used for ED is available if he is interested. c. there are many methods of sexual expression that can be alternatives to sexual intercourse. d. sterility will not be a problem after surgery because sperm production will not be affected.

a

The nurse has obtained the health history, physical assessment data, and labora-tory results shown in the accompanying figure for a patient admitted with aplastic anemia. Which information is most important to communicate to the health care provider? History Physical Assessment Laboratory Results • Fatigue, which has increased over last month • Frequent constipation • Conjunctiva pale pink, moist • Multiple bruises • Clear lung sounds • Hct 33% • WBC 1500/µL • Platelets 70,000/µL a. Neutropenia c. Increasing fatigue b. Constipation d. Thrombocytopenia

a

The nurse is caring for a client diagnosed with asthma. While performing the shift assessment, the nurse auscultates breath sounds including sibilant wheezes, which are continuous musical sounds. What characteristics describe sibilant wheezes? a) They can be heard during inspiration and expiration. b) They result from air passing through widened air passages. c) They are heard in clients with decreased secretions. d) They occur when the pleural surfaces are inflamed.

a

The nurse is caring for a client with chronic obstructive pulmonary disease. The client calls the doctor and states having difficulty breathing and overall feeling fatigued. The nurse realizes that this client is at high risk for which condition? a) Respiratory acidosis b) Metabolic alkalosis c) Metabolic acidosis d) Respiratory alkalosis

a

When assessing a patient with chronic heart failure, the healthcare provider would expect to identify which of these clinical manifestations? A) Inspiratory crackles B) Asymmetrical Chest Expansion C) Expiratory Wheezing D) Subcutaneous crepitus

a

Which patient should the nurse assign as the roommate for a patient who has aplastic anemia? a. A patient with chronic heart failure b. A patient who has viral pneumonia c. A patient who has right leg cellulitis d. A patient with multiple abdominal drains

a

1. A nurse is reviewing the plan of care for a client who has systemic lupus erythematosus (SLE). The client reports fatigue, joint tenderness, swelling, and difficulty urinating. Which of the following laboratory findings should the nurse anticipate? (Select all that apply.) A. Positive ANA titer B. Increased hemoglobin C. 2+ urine protein D. Increased serum C3 and C4 E. Elevated BUN

a, c, e

A pt with COPD is likely to have which findings on assessment? Select All That Apply A. increased AP diameter of chest B. sitting in a chair leaning forward with elbows on knees C. unintentional weight gain D. decreased appetite E. unexplained weight loss

a,b,d,e

6) A nurse is caring for a client with systemic lupus erythematosus (SLE). The client begins to cry and tells the nurse that she is afraid that her skin will be disfigured with lesions. Which intervention does the nurse plan to teach this client to minimize skin infections associated with SLE? Select all that apply. A) Use sunscreen with an SPF of 15 or greater. B) Remain indoors on sunny days. C) Avoid swimming in a pool or the ocean. D) Avoid sun exposure between 10:00 a.m. and 3:00 p.m. E) Decrease sun exposure between 3:00 p.m. and 5:00 p.m.

a,d

A nurse is assessing the neurovascular status of a client who returned to the surgical nursing unit 4 hours ago after undergoing aortoiliac bypass graft. The affected leg is warm, and the nurse notes redness and edema. The pedal pulse is palpable and unchanged from admission. The nurse interprets that the neurovascular status is: a. Normal because of the increased blood flow through the leg b. Slightly deteriorating and should be monitored for another hour c. Moderately impaired, and the surgeon should be called. d. Adequate from the arterial approach, but venous complications are arising.

a. An expected outcome of surgery is warmth, redness, and edema in the surgical extremity because of increased blood flow. Options 2, 3, and 4 are incorrect interpretations.

Signs and symptoms of sleep apnea include which of the following?a. Loud snoring b. Difficulty falling asleep c. Headache in the evening d. Nighttime sleepiness

a. Loud snoring

Which collaborative problem will the nurse include in a care plan for a patient admitted to the hospital with idiopathic aplastic anemia? a. Potential complication: seizures b. Potential complication: infection c. Potential complication: neurogenic shock d. Potential complication: pulmonary edema

b

A nurse is caring for a client who had a percutaneous insertion of an inferior vena cava filter and was on heparin therapy before surgery. The nurse would inspect the surgical site most closely for signs of: a. Thrombosis and infection b. Bleeding and infection c. Bleeding and wound dehiscence. d. Wound dehiscence and evisceration.

b. After inferior vena cava insertion, the nurse inspects the surgical site for bleeding and signs and symptoms of infection. Otherwise, care is the same as for any post-op client

A client comes to the outpatient clinic and tells the nurse that he has had legs pains that begin when he walks but cease when he stops walking. Which of the following conditions would the nurse assess for? a. An acute obstruction in the vessels of the legs b. Peripheral vascular problems in both legs c. Diabetes d. Calcium deficiency

b. Intermittent claudication is a condition that indicates vascular deficiencies in the peripheral vascular system. If an obstruction were present, the leg pain would persist when the client stops walking. Low calcium levels may cause leg cramps but would not necessarily be related to walking.

A 24-year old man seeks medical attention for complaints of claudication in the arch of the foot. A nurse also notes superficial thrombophlebitis of the lower leg. The nurse would next assess the client for: a. Familial tendency toward peripheral vascular disease b. Smoking history c. Recent exposures to allergens d. History of insect bites

b. The mixture of arterial and venous manifestations (claudication and phlebitis, respectively) in the young male client suggests Buerger's disease. This is an uncommon disorder characterized by inflammation and thrombosis of smaller arteries and veins. This disorder typically is found in young adult males who smoke. The cause is not known precisely but is suspected to have an autoimmune component.

A patient is admitted to the hospital with a diagnosis of abdominal aortic aneurysm. Which signs and symptoms would suggest that his aneurysm has ruptured? a.Sudden shortness of breath and hemoptysis b.Sudden, severe low back pain and bruising along his flank c.Gradually increasing substernal chest pain and diaphoresis d.Sudden, patchy blue mottling on feet and toes and rest pain

b.Sudden, severe low back pain and bruising along his flank The clinical manifestations of a ruptured abdominal aortic aneurysm include severe back pain, back or flank ecchymosis (Grey Turner's sign), and hypovolemic shock (tachycardia, hypotension, pale clammy skin, decreased urine output, altered level of consciousness, and abdominal tenderness).

A pt has COPD with chronic difficult breathing. In planning this pts care, what condition must the nurse acknowledge is present in this pt? A. decreased need for calories and protein requirements since dyspnea causes activity intolerance B. COPD has no effect on calorie and protein needs, meal tolerance or appetite C. increased metabolism and the need for additional calories and protein supplements D. anabolic state, which creates conditions for building body strength and muscle mass

c

During an assessment of a 45-year-old patient with asthma, the nurse notes wheezing and dyspnea. The nurse interprets that these symptoms are related to what pathophysiologic change? a. Laryngospasm b. Pulmonary edema c. Narrowing of the airway d. Overdistention of the alveoli

c

Which of the following characteristics is typical of the pain associated with DVT? a. Dull ache b. No pain c. Sudden onset d. Tingling

c. DVT is associated with deep leg pain of sudden onset, which occurs secondary to the occlusion. A dull ache is more commonly associated with varicose veins. A tingling sensation is associated with an alteration in arterial blood flow. If the thrombus is large enough, it will cause pain.

In a client with Amyotrophic Lateral Sclerosis (ALS), difficulty in swallowing or chewing or dysphagia can happen at any point. The nurse knows that the following are appropriate actions to prevent aspiration, except? a. Assessing gag reflex before giving food or fluid by mouth b. Positioning the client in an upright position during mealtime and for ½ afterward c. Having a client sip through a straw when drinking while lying down d. None of the above

c. Having a client sip through a straw when drinking while lying down

a client has a NG tube following a subtotal gastrectomy. the nurse should: a. irrigate the tube with 30 mL of sterile water every hour, if needed b. reposition the tube if it is not draining well c. monitor the client for nausea, vomiting, and abdominal distention d. turn the machine to high suction if the drainage is sluggish on low suction

c. monitor the client for nausea, vomiting, and abdominal distention

A 50-year-old woman weighs 95 kg and has a history of tobacco use, high blood pressure, high sodium intake, and sedentary lifestyle. When developing an individualized care plan for her, the nurse determines that the most important risk factors for peripheral artery disease (PAD) that need to be modified are a.weight and diet. b.activity level and diet. c.tobacco use and high blood pressure. d.sedentary lifestyle and high blood pressure.

c. tobacco use and high blood pressure Significant risk factors for peripheral artery disease include tobacco use, hyperlipidemia, elevated levels of high-sensitivity C-reactive protein, diabetes mellitus, and uncontrolled hypertension; the most important is tobacco use. Other risk factors include family history, hypertriglyceridemia, hyperuricemia, increasing age, obesity, sedentary lifestyle, and stress.

During assessment of a pt with obstructive jaundice, the nurse would expect to find: clay colored stools dark urine and stool pyrexia and pruritis elevated urinary urobilinogen

clay colored stools

A nurse is providing dietary instructions to a client with hypoglycemia. To control hypoglycemic episodes, the nurse should recommend: increasing saturated fat intake and fasting in the afternoon. eating a candy bar if light-headedness occurs. increasing intake of vitamins B and D and taking iron supplements. consuming a low-carbohydrate, high-protein diet and avoiding fasting.

consuming a low-carbohydrate, high-protein diet and avoiding fasting.

A patient undergoing a TURP returns from surgery with a three-way urinary catheter with continuous bladder irrigation in place. The nurse observes that the urine output has decreased and the urine is clear red with multiple clots. The patient is complaining of painful bladder spasms. The most appropriate action by the nurse is to a. administer the ordered IV morphine sulfate, 4 mg. b. increase the flow rate of the continuous bladder irrigation. c. give the ordered the belladonna and opium suppository. d. manually instill 50 ml of saline and try to remove the clots.

d

An older adult patient experiences an asthma attack that is severe enough to warrant the use of a rescue drug. Which medication is best to use for the acute symptoms? A. Omalizumab (Xolair) B. Fluticasone (Flovent) C. Salmeterol (Serevent) D. Albuterol (Proventil)

d

a client who is recovering from a subtotal gastrectomy experiences dumping syndrome. the client asks the nurse, "when will i be able to eat three meals a day again like i used to?" which of the following response by the nurse is MOST appropriate? a. "eating six meals a day is time-consuming, isn't it?" b. "you will have to eat six meals a day for the rest of your life." c. "you will be able to tolerate three meals a day before you are discharged." d. "most clients can resume their normal meal patterns in about 6-12 months"

d. "most clients can resume their normal meal patterns in about 6-12 months"

A client with Amyotrophic Lateral Sclerosis can have weakened articulation, in this case, the nurse should do the following communication strategies, except: a. Sit face to face with the client in a quiet, well lit room and limit the amount of distraction. b. Repeat the words that are understood to validate with the client. c.Establish the type of communication environment that is comfortable for the client. d. Finish the client's sentences for him and encourage him to say his ideas fast.

d. Finish the client's sentences for him and encourage him to say his ideas fast.Hurrying the client does not help him articulate his ideas better. The nurse should have the client with ALS slow down and take his time when talking.

A client is admitted with a venous stasis leg ulcer. A nurse assesses the ulcer, expecting to note that the ulcer: a. Has a pale colored base b. Is deep, with even edges c. Has little granulation tissue d. Has brown pigmentation around it.

d. Venous leg ulcers, also called stasis ulcers, tend to be more superficial than arterial ulcers, and the ulcer bed is pink. The edges of the ulcer are uneven, and granulation tissue is evident. The skin has a brown pigmentation from accumulation of metabolic waste products resulting from venous stasis. The client also exhibits peripheral edema. (options 1, 2, and 3 is due to tissue malnutrition; and thus us an arterial problem)

to reduce the risk of dumping syndrome, the nurse should teach the client to do which of the following? a. sit upright for 30 minutes after meals b. drink liquids with meals, avoiding caffeine c. avoid milk and other dairy products d. decrease the carb content of meals

d. decrease the carb content of meals

A client tells the nurse that she has been working hard for the past 3 months to control her type 2 diabetes with diet and exercise. To determine the effectiveness of the client's efforts, the nurse should check: urine glucose level. serum fructosamine level. glycosylated hemoglobin level. fasting blood glucose level.

glycosylated hemoglobin level.

During assessment of a client with amyotrophic lateral sclerosis (ALS), which finding should the nurse identify when planning care for this client? A) Muscle weakness. B) Urinary frequency. C) Abnormal involuntary movements. D) A decline in cognitive function.

muscle weakness

A child is having an asthma attack and the parent can't remember which inhaler to use for quick relief. The nurse accesses the child's medication information and tells the parent to use which inhalant? -Cromolyn sodium -Theo-Dur -Serevent -Proventil

proventil

The nurse is assigned to care for a patient in the emergency department admitted with an exacerbation of asthma. The patient has received a β-adrenergic bronchodilator and supplemental oxygen. If the patient's condition does not improve, the nurse should anticipate what as the most likely next step in treatment? IV fluids Biofeedback therapy Systemic corticosteroids Pulmonary function testing

systemic corticosteroids


Conjuntos de estudio relacionados

Ch. 11 Information and Data Management: Organizing, Verifying, Maintaining, and Accessing

View Set

BIOL 1031 Mastering Biology Chapter 50

View Set

Week 9: Chapter 51 Diuretic Agents

View Set

The 18th Century and the Enlightenment

View Set

The Art of Public Speaking - Final Exam

View Set

Medical Terminology: Chapters 1, 2, 3 - Quiz 1

View Set

Dermatological, Ophthalmic, and Otic Drugs

View Set